ABPN Exam Review Terms
- In 2012 what did the National Institute on Drug Abuse (NIDA), say about methadone treatment?
"For methadone maintenance, 12 months of treatment is the minimum, and some opiate-addicted individuals will continue to benefit from methadone maintenance treatment over a period of years."
- The Z-drugs and most Benzodiazepine metabolized primarily by the CYP3A4 enzyme.
"but a LOT dont" lorazepam, oxazepam, and tamazepam) do not go through cytochrome P450 metabolism (Phase I metabolism), and are only metabolized via glucuronidation (Phase II metabolism) Potent inhibitors of CYP3A4 include clarithromycin, erythromycin, diltiazem, itraconazole, ketoconazole, ritonavir, verapamil, goldenseal and grapefruit. Inducers of CYP3A4 include carbamazepin, phenobarbital, phenytoin, rifampicin, St. John's Wort and glucocorticoids.
- Amobarbital is used when?
'use the Amo for Allergy' Amobarbital for agitation & allergy to benzo/or phenobarbital.
What is the treatment of choice in patients with antisocial personality disorder who show impulsive behavior or ictal aggression with abnormal EEG findings?
- Antiepileptic drugs like carbamazepine and valproate
What is used to treat dementia with Lewy Bodies?
- Cholinesterase inhibitors: Rivastigmine, Donepezil, and Galantamine are commonly used and can be used for dementia with Lewy Bodies.
what is depression called and timing when in or after pregnancy?
- DSM V indicates that onset within 4 weeks of delivery must occur for the diagnosis of major depressive disorder "with peripartum onset" which is defined as the most recent episode occurring during pregnancy as well as in the four weeks following delivery.
A 24-year-old female is brought to the ER 1 hour after acute onset of double vision in the right eye. Two months ago, she had an acute episode of pain with movement of the left eye along with loss of both central vision and pupillary light reflex. These symptoms subsided spontaneously. Which of the following medications has been shown to be the most effective at decreasing the progression of the relapsing-remitting form of this disease?
- Glatiramer acetate Is the one medication approved for relapsing-remitting MS.
What is the factor if medication will/won't cross the blood-brain barrier?
- Lipid solubility is the determining factor
What is used to treat Huntington's Disease?
- Tetrabenazine can be as supportive therapy. a reversible VMAT-2 inhibitor, high-affinity inhibitor of mono-amine uptake into granular vesicles of presynaptic neurons by binding selectively to VMAT-2. As a result of this inhibition, monoamine degradation in the neuron is augmented, leading to depletion of the monoamines, particularly dopamine.
- What are the Three diagnostic criteria for neurocognitive disorder due to traumatic brain injury?
-First is that the criteria should be met for mild or major neurocognitive disorder which is met by this patient's neurocognitive symptoms. 2. evidence of traumatic brain injury by the presence of any one of the following: loss of consciousness, posttraumatic amnesia, disorientation and confusion and presence of injury on neuroimaging with or without neurological deficits.
18-year-old woman presents with a two year history of anxiety, gait instability, and progressively worsening tics. She has no family history of neurologic disease, but her father committed suicide at age 32. MRI scan of the brain shows atrophy of the cerebral cortex and caudate nucleus, what is dx?
-Juvenile Huntington's disease is a rare disorder occurring before the age of twenty years. It is commonly seen with autosomal dominant inheritance from the father. -m/f effected equally Autosomal dominant CAG triplet repeat expansion in one allele of the HD (huntingtin) gene on chromosome 4 Number of CAG repeats determines presence of disease < 29 repeats: no disease, or normal phenotype 30-35 repeats: intermediate (no risk to the individual, but increased risk for offspring) allele 36-39 repeats: incomplete penetrance allele > 39 repeats: full penetrance allele Number of CAG repeats may increase with subsequent generations,(anticipation) particularly with paternal transmission HD mutation results in neuronal death, particularly in the caudate nucleus and putamen (basal ganglia) Age of onset and severity of disease is inversely correlated with number of CAG repeats Supportive treatment can be highly effective reserpine, tetrabenazine, or atypical antipsychotics for involuntary movements SSRIs for depression atypical antipsychotics for psychosis and behavior problems benzodiazepines for sleep disturbance Genetic counseling should be offered for patients, family members, and at-risk offspring
What disease modifying agent is approved for PPMS (primary progressive multiple sclerosis)?
-No disease-modifying agent is approved for treatment of
- What are Poor prognosticators of cerebral anoxia are duration of anoxia?
-longer than 8 to 10 minutes -duration of CPR longer than 30 minutes -duration of postanoxic coma longer than 72 hours, -pupillary light reaction absent on day 3, -motor response to pain absent on day 3, -Glasgow coma scale on day 3 less than 5, -brain stem reflexes absent, -blood glucose of greater than 300 mg/dl.
RRR
1 - RR
NNH
1 / AR
NNT
1 / ARR
- The prevalence of schizophrenia is?
1% in the general population 8% in non-twin siblings of a schizophrenic patient, 12% in children with 1 schizophrenic parent, 12% in dizygotic twins of schizophrenic patients, 40% in children with 2 schizophrenic parents, 47% in monozygotic twins of a schizophrenic patient.
Freud's Psychosexual Stages
1. Oral Stage 0-1yo 2. Anal Stage 1-3yo 3. Phallic Stage 3-6yo (Oedipal/Electra complex)- masturbation/penis envy 4. Latency Stage 6-12yo (intellectual interest, sexual repressed) 5. Genital Stage 12-18yo
- What are the common reasons for noncompliance with medications:
1. Patients cannot afford them 2. Side effects are intolerable 3. Psychiatric symptoms interfere with treatment (e.g. delusions of being poisoned) 4. The instructions are poorly given 5. Patients like their symptoms and don't want them treated (e.g. patients prefer manic states) 6. The lives of some patients are so chaotic and disorganized that good compliance is difficult without close monitoring and follow-up
Piaget's Stages
1. sensorimotor- Object permanence 0-2yo 2. preoperational- parallel play 2-7yo 3. concrete operational- Conservation (math/measurements) 7-11 yo 4. formal operational- Abstract/ hypothetical logic 12- adulthood
- What percent of patients develop benign rash on Lamotrigine?
10% of patients started on lamotrigine develop a benign rash
- How long before starting Disulfiram?
12-24 hours after last drink is usually enough to clear all alcohol from the system and allow initiation of disulfiram.
- Paroxysmal hemicranias (women
2-30 mins many times a day); Tx: indomethacin
- 3
4-Dihydroxyphenylacetic acid (DOPAC) and homovanillic acid (HVA) are both primary metabolites of dopamine that may be useful in identifying the levels of dopaminergic activity.
- Buspirone acts on what receptors?
5HT-1A receptors.
- Cluster headaches (men
>30 mins): verapamil for PPX; oxygen, triptans, steroids for abortive
Small insertions, deletions, or duplications within the DNA molecule are referred to as: A. point mutations. B. nonsense mutations. C. chromatin remodeling. D. copy number variations. E. single nucleotide polymorphisms.
A A. point mutations.- change the gene B. nonsense mutations.- Stop mutation terminates the gene C. chromatin remodeling. D. copy number variations. E. single nucleotide polymorphisms. Neurosciences
The psychiatrist is asking a patient questions describing symptoms of mania. The patient responds, "that sounds exactly like my cousin," and proceeds to tell a long story about the cousin impulsively traveling to Mexico and exhausting all savings, never actually answering the psychiatrist's request. This is an example of: A. tangentiality. B. circumstantiality. C. flight of ideas. D. pressured speech. E. clang associations.
A A. tangentiality. B. circumstantiality. - tangential but eventually come back and answer the Q. C. flight of ideas. - tangential but more disorganized. D. pressured speech. E. clang associations. Clinical psychiatry Diagnostic procedures
A 25-year-old patient is evaluated for daytime sleepiness. The patient reports falling asleep at night without difficulty and waking up at 7 a.m. every morning. The patient's spouse indicates that the patient does not snore. During the day the patient needs to take several brief naps which come on suddenly, and has fallen asleep while driving on one occasion. Sleep study reveals decreased REM latency. Which of the following would be the most effective treatment? A. Modafinil B. Zolpidem C. Oral appliance D. Avoid caffeine E. Continuous positive airway pressure
A Clinical neurology
A 32-year-old patient presents to the emergency department with three days of recurrent headaches. The pain is described as sudden in onset, predominantly periorbital and frequently awakening the patient from sleep. The episodes last for approximately 40 minutes and spontaneously resolve. On examination the patient appears restless, has mild ptosis of the left eye, mild left miosis, increased lacrimation, nasal congestion and rhinorrhea all ipsilateral to the pain. The examination is otherwise unremarkable. Which of the following treatments is most likely to abort the headache? A. Oxygen B. Ibuprofen C. Topiramate D. Indomethacin E. Dexamethasone
A Clinical neurology
A 55-year-old patient complains of "the worst headache of my life" and is evaluated for subarachnoid hemorrhage. Computed tomography (CT) scan of the brain is normal and does not show any blood. Which of the following is the most appropriate next step to determine the presence of subarachnoid blood? A. Lumbar puncture B. Conventional angiogram C. Outpatient follow up in one week D. Repeat CT scan of the brain in 24 hours E. Magnetic resonance imaging (MRI) scan of the brain
A Clinical neurology
A magnetic resonance imaging (MRI) scan of the head reveals an infarct in the distribution of the left anterior cerebral artery, Which of the following is the patient most likely to exhibit? A. Weakness of the contralateral foot and leg B. Incoordination of the ipsilateral arm and leg C. Visual field deficit consisting of homonymous hemianopia D. Ipsilateral third nerve palsy and contralateral hemiplegia E. Loss of pinprick sensation in the ipsilateral face and contralateral arm and leg
A Clinical neurology
A patient in the movement disorder clinic is evaluated for complaints of involuntary movements in the upper extremities which are described as sudden jerks, sometimes preceded by an urge to move. The movements are followed by a sense of relief. These movements are best characterized as: A. tics. B tremors. C. myoclonus. D. stereotypies. E. compulsions.
A Clinical neurology
Which of the following tests assesses for ideomotor apraxia in a patient with intact language comprehension? A. Asking the patient to pretend to light a match B. Giving the patient a pen and observing how he uses it C. Asking the patient to copy a picture of intersecting pentagons D. Having the patient tap his thumb and index finger together quickly E. Asking the patient to describe the steps needed to brush his teeth
A Clinical neurology
Federal courts have ruled that which of the following is a required component of psychiatric care in correctional institutions? A. Suicide prevention program B. Substance abuse counseling C. Participation by forensic psychiatrists D. Routine use of restraints and seclusion
A Clinical psychiatry Administration & systems
The operant procedure in which a desirable behavior pattern is learned by the successive reinforcement of approximations to that behavior is called: A. shaping. B. modeling. C. habituation. D. conditioning. E. sensitization
A Clinical psychiatry Behavioral & social sciences
Maternal smoking is known to be associated with which of the following effects on fetal physical development? A. Decreased birth weight B. Increased head circumference C. Accelerated lung development D. Decreased incidence of cleft palate E. Increased incidence of cardiac malformations
A Clinical psychiatry Development & maturation
Which of the following is the most sensitive and specific laboratory test for identifying heavy drinking and monitoring treatment? A. Carbohydrate deficient transferrin (CDT) B. Mean corpuscular volume (MCV) C. Alkaline phosphatase (ALP) D. Alanine aminotransferase (ALT) E. Aspartate aminotransferase (AST)
A Clinical psychiatry Diagnostic procedures
A patient presents to the psychiatrist requesting an evaluation for dementia. He reports that his father had vascular dementia, and he wants to prevent a similar fate for himself. His past medical history is significant for diabetes, hypertension, and hyperlipidemia. On evaluation, there is no evidence of significant memory impairment or dementia. The psychiatrist recommends that the patient follow up with his primary care physician to optimize management of blood pressure, diabetes, and lipids. This recommendation is an example of which of the following? A. Primary prevention B. Secondary prevention C. Tertiary prevention D. Universal intervention E. Acute intervention
A Clinical psychiatry Epidemiology
Children in which of the following age groups are at greatest risk for death resulting from child abuse? A. Birth - 3 years B. 4 - 6 years C. 7 - 10 years D. Il - 13 years E. 14 - 18 years
A Clinical psychiatry Epidemiology
In the Clinical Antipsychotic Trials of Intervention Effectiveness (CATIE) study, 60% of patients with schizophrenia reported some substance abuse (not including nicotine). Which of the following substances was most commonly used? A. Alcohol B. Cocaine C. Marijuana D. Stimulants E. Methamphetamines
A Clinical psychiatry Epidemiology
Which of the following is a risk factor for child abuse? A. Financial stress B. Small family size C. Two-parent household D. Child older than 13 years E. College educated parents
A Clinical psychiatry Epidemiology
An outpatient clinic requires patients to show two pieces of identification before every appointment. This policy helps ensure that all care provided is: A. safe. B. timely. C. efficient. D. effective. E. patient-centered
A Clinical psychiatry Issues in practice
Which of the following is an aspect of legal standards for competence? A. The law presumes a person is competent. B. Competence must be assessed before each decision. C. The law presumes a person with dementia is incompetent. D. The law presumes a person with active psychosis is incompetent. E. The law presumes a person with developmental delay is incompetent.
A Clinical psychiatry Issues in practice
A patient presents to the primary care clinic with concern about having a brain tumor after an acquaintance died from brain cancer. The patient denies any headache or other neurologic symptoms. However, the patient reports reading on the internet that brain tumors can be easily missed and that there may not be any obvious symptoms initially. There are no neurologic or cognitive findings on exam and a computed tomography (CT) scan is normal. However, the patient returns a month later insisting that other tests such as a brain positron emission tomography (PET) scan be done. The patient has been worried about a missed diagnosis, and has continued to spend hours researching brain tumors. The patient displays no other psychiatric symptoms. Which of the following is the patient's most likely diagnosis? A. Illness anxiety disorder B. Somatic symptom disorder C. Adjustment disorder with anxiety D. Delusional disorder, somatic type E. Functional neurological symptom disorder
A Clinical psychiatry Psychopathology & associated conditions
A patient who recently delivered a full term newborn is brought to the emergency department by her husband. The patient is agitated, labile, and demanding to leave the hospital because she says that her baby is being poisoned. This condition is considered a psychiatric emergency because of its association with: A. infanticide. B. bipolar disorder. C. substance abuse. D. psychotic disorder. E. high recurrence rate.
A Clinical psychiatry Psychopathology & associated conditions
Which of the following interventions has the most robust evidence for preventing suicide? A. Restricting access to lethal methods B. Media reporting guidelines for suicide C. Public education campaigns about suicide D. Screening interventions within high schools E. Prescription of selective SSRIs for those at high risk
A Clinical psychiatry Psychopathology & associated conditions
Which of the following is a paramnesia? A. Déjå vu B. Melancholia C. Folie å deux D. La belle indifference E. Pseudologia phantastica
A Clinical psychiatry Psychopathology & associated conditions
In addition to atypical antipsychotics, which of the following augmenting agents is most effective in depressed patients with insufficient response to antidepressant monotherapy? A. Lithium B. Valproate C. Lamotrigine D. Levothyroxine E. Methamphetamine
A Clinical psychiatry Treatment
The dialectical behavioral therapist primarily focuses on which of the following? A. Utilizing contingencies and skills to replace the dysfunctional behavior B. Creating a holding environment to analyze the cause of dysfunctional behavior C. Encouraging expression of intense affect and encouraging disruptive behavior D. Facilitating expression of intense feelings, providing support for these feelings, and using gradual tasks to substitute for agitation
A Clinical psychiatry Treatment
The efficacy of case management in the treatment of substance use problems is best established for which of the following outcomes? A. Retention in treatment B. Enhanced quality of life C. Reduced substance use D. Lower rates of HIV infection E. Improved occupational function
A Clinical psychiatry Treatment
Which of the following types of therapies is generally most appropriate for patients who have poor reality testing, primitive and immature defenses, Inadequate affect regulation and/or poor impulse control? A. Supportive B. Interpersonal C. Psychoanalytic D. Short-term psychodynamic E. Acceptance and commitment
A Clinical psychiatry Treatment
A two-year-old child presents for routine physical exam. Which of the following skills is the child likely to be able to demonstrate? A. Kick a ball B. Catch a ball C. Copy a circle D. Ride a tricycle E. Hop on one foot
A Neurosciences
Abnormal emotional expression such as pathological laughter or crying is most likely caused by lesions affecting cortical-subcortical circuits linking the frontal cortex, pons and which of the following? A. Cerebellum B. Dentate gyrus C. Raphe nucleus D. Intralaminar thalamic nuclei E. Nucleus of the trigeminal nerve
A Neurosciences
Allelic exclusion and X-chromosome inactivation can be best explained by which of the following mechanisms? A. Epigenetic B. Post-translational C. Somatic mutation D. Post-transcriptional E. Environmental influences
A Neurosciences
Potentially threatening objects produce a startle response before a person becomes consciously aware of them. This process is due to direct connections of the thalamus with which of the following? A. Amygdala B. Hippocampus C. Primary visual cortex D. Medial prefrontal cortex E. Autonomic nervous system
A Neurosciences
Which of the following developmental disorders is a trinucleotide repeat disorder? A. Friedreich ataxia B. Klinefelter syndrome C. Turner syndrome D. Williams syndrome E. Angelman syndrome
A Neurosciences
Which of the following is the gene mapping strategy that involves identification of genetic markers that co-segregate with disease phenotype? A. Pedigree analysis B. Affected sib-pair analysis C. Family trios genome wide association D. Case control genome wide association
A Neurosciences
Which of the following conditions is most likely to be associated with the finding of upbeating nystagmus on primary gaze on neurologic exam? A. Medullary lesion B. Pontine hemorrhage or infarct C. Occult lung or breast carcinoma D. Craniopharyngioma in the optic chiasm region E. Congenital anomaly of the anterior third ventricle
A "Medullary's Melody is so Upbeat" Upbeat: Upbeat nystagmus is a type of central vestibular nystagmus. It can be caused by lesions from the medulla to the midbrain. It usually increases in up gaze, but not on lateral gaze, and fixation does not dampen it. Causes of upbeat nystagmus include: cerebellar degeneration, multiple sclerosis, infarction of the medulla, tumours, Wernicke's encephalopathy, brain stem encephalitis, Behcet's syndrome, meningitis, congenital, middle ear disease, and drugs. This condition is often seen with medullary lesions Downbeat: cerebellar and spinocerebellar degeneration syndromes, MS and developmental anomalies affecting the pons and cerebellum. Neurosciences
- What is taper duration for Lithium?
A two-week taper is the minimum for discontinuation of lithium.
The "false belief' test of young children uses two dolls in an interaction whereby one doll hides a marble in front of a second doll, and then moves the marble while the second doll leaves the scene. The second doll re-enters the scene and the child subject is then asked where this doll believes the marble to be. This test is used to assess which of the following? A. Theory of mind B. Imaginary play C. Conventional morality D. Verbal communication E. Nonverbal communication
A. Theory of Mind theory of mind refers to the capacity to understand other people by ascribing mental states to them Clinical psychiatry Development & maturation
- Tics/Tourette's:
Abilify, pimozide, Haldol
- What do ACE-inhibitors and Thiazides do to Lithium?
Ace, Thiazides Increase lithium levels.
How does Linezolid improve depression?
An oxazolidinone antibiotic -originally discovered as a psychotropic agent with antidepressant effects through mild reversible nonselective inhibition of monoamine oxidase inhibitors. Linezolid is used to treat infections, including pneumonia, and infections of the skin . Linezolid is in a class of antibacterials called oxazolidinones. It works by stopping the growth of bacteria. Antibiotics such as linezolid will not work for colds, flu, and other viral infections.
- When aluminum hydroxide is taken concurrently it can do what?
Antacids are known for affecting the absorption of other drugs causing even clinically significant alterations on the drug's effects. it can affect the absorption of the following drugs: diflunisal, digoxin, ferrous ions, ketoconazole, tetracyclines, quinolone antibiotics, aminophylline, diazepam, levodopa, phenothiazines (for example chlorpromazine), and phenytoin.
- The anti-depressant effects of mirtazapine are largely due to?
Antagonism of central presynaptic Alpha-2 adrenergic receptors which causes increased firing of norepinephrine and serotonin neurons. Antagonism of Alpha-1 adrenergic blockade helps in relieving anxiety and makes this drug useful in the treatment of PTSD. Antagonism of 5HT2 and 5HT3 receptors but this antagonism is related to its use as an anxiolytic and hypnotic drug. 5HT3 receptive blockade is also used to combat serious GI effects of cancer chemotherapy. These receptors antagonism also leads to appetite stimulation. Antagonism of histamine H1 receptors in lower doses, which causes increased somnolence.
- Patient brought into ED by family originally from Southeast and South Asia, and parts of east Africa and Betel nuts are chewed in some parts of the world to produce a mild euphoria and sensation of floating in space.
Areca nut affects almost all organs of the human body, including the brain, heart, lungs, gastrointestinal tract and reproductive organs. It causes or aggravates pre-existing conditions such as neuronal injury, myocardial infarction, cardiac arrhythmias, hepatotoxicity, asthma, central obesity, type II diabetes, hyperlipidemia, metabolic syndrome, etc. Areca nut affects the endocrine system, leading to hypothyroidism, prostate hyperplasia and infertility. It affects the immune system leading to suppression of T-cell activity and decreased release of cytokines. It has harmful effects on the fetus when used during pregnancy.
- Organophosphate pesticides are powerful acetylcholinesterase inhibitors. Poisoning leads to excessive cholinergic activity and SLUDGE signs (salivation, lacrimation, urination, diaphoresis (or defecation), gastrointestinal motility, and emesis). What is treatment?
Atropine, which is a muscarinic acetylcholine receptor antagonist, is an antidote.
Mahler's Phases
Autistic First weeks of life Total detachment and self absorption; mostly sleeping Symbiotic First weeks - 5 mos Recognizes mother but lacks individuality; NPD due to inadequate soothing and inadequate refueling during separation-individuation Separation-individuation Hatching - increased interest in outside world Practicing (9-16 mos) - develops physical ability to separate from mother Rapproachment (15-24 mos) - infant explores the outside world but requires mother to be present for emotional support -BPD due to disturbance in rapproachment
What medication would an Azole antifungals do to diazepam serum levels?
Azoles inhibit hepatic oxidative metabolism via CYP 3A4. This leads to an increase of diazepam serum levels as opposed to induction of CYP 3A4 which would cause a decrease in benzodiazepine level.
Which of the following is a core feature of factitious disorder? A. Symptoms must be present for at least one month B. Intentional falsification of medical or psychological symptoms C. Association of an obvious external reward with the illness behavior D. Symptoms are incompatible with known medical or neurological illness E. Persistence of belief that one is ill when presented with evidence to the contrary
B Factitious disorder imposed on self (Munchausen syndrome) the patient falsifies physical or psychological symptoms or induces injury or disease to themselves. when in another person (e.g., a child) it is termed factitious disorder imposed on another (Munchausen syndrome by proxy) Vulnerable child syndrome when seen in parents of a child who has a history of being ill followed by excess concern for minor concerns. the patient's deceptive behavior occurs in the absence of external rewards this disorder is not better explained by another mental disorder (e.g., delusional disorder) Treatment no specific therapy in factitious disorder imposed on another (e.g., in a child) children must be removed by child protective services Clinical psychiatry Psychopathology & associated conditions
Which of the following second generation antipsychotic medications carries package labeling by the FDA highlighting the risk of QTc prolongation? A Quetiapine B Paliperidone C Aripiprazole D Olanzapine E Clozapine
B "Dones- Paliperidone and Ziprazidone" Clinical psychiatry Treatment
A 60-year-old man with a history of liver disease presents with confusion. On examination the patient demonstrates asterixis of the hands and is found to have an elevated serum ammonia level. Electroencephalograph (EEG) is likely to demonstrate which of the following? A. Normal alpha rhythm B. Bilateral triphasic waves C. Focal slowing in the temporal lobe D. Continuous spike wave discharges E. Periodic lateralized epileptiform discharges
B Clinical neurology
Inability to name objects held in the left hand, but not in the right hand, indicates a lesion in which of the following? A Left frontal lobe B CoIpus callosum C Left parietal lobe D Right parietal lobe E Anterior commissure
B Clinical neurology
Use of mental health courts for qualified defendants has been shown to increase which of the following? A. Remorse for criminal acts B. Engagement in community treatment C. The number of inmates treated in prison D. Long-term abstinence from illicit substances E. Overall cost to the justice and mental health system
B Clinical psychiatry Administration & systems
A patient presents to the psychiatrist for evaluation of mental and physical exhaustion, lightheadedness, difficulty concentrating, headaches, and gastrointestinal distress. The symptoms have been present for about 6 months and first began when the patient immigrated to the United States from China, Since arriving in the U.S., the patient has struggled to find work and has been living with relatives. In assessing this patient's symptoms, which of the following questions is most appropriate? A. "Do you think you are depressed?" B. "What do you think is causing your symptoms?" C. "Do you know anyone else with these symptoms?" D. "On a scale of 1-10, how severe is your exhaustion?" E. "Are you willing to take medications to help you feel better?"
B Clinical psychiatry Behavioral & social sciences
According to Winnicott, a "good enough" parent is attuned to the infant's mental state most of the time. The parent communicates this understanding to the infant via which of the following processes? A. Holding B. Mirroring C. Projecting D. Interpreting E. Internalizing
B Clinical psychiatry Behavioral & social sciences
In behavioral psychology, extinction is defined as: A. strengthening behavior by removing an unpleasant event. B. weakening of the conditioned response due to withdrawal of the stimulus. C. diminishing the probability of a behavior by pairing it with an aversive event. D. repeated pairing of a neutral stimulus with one that evokes an emotional response. E. decreasing strength of the association between stimulus and response due to repetition of the stimulus.
B Clinical psychiatry Behavioral & social sciences
At what age does a typically developing child usually first speak in complete sentences? A. 2 years B. 3 years C. 4 years D. 5 years E. 6 years
B Clinical psychiatry Development & Maturation
Third- and fourth-grade students from several different classrooms are sharing the school playground. Which of the following is likely to be the most dominant observable social pattern? A. Boy-girl pairing for role-playing games B. Same-sex groupings regardless of activity C. Initiation of boy-girl, mixed-grade games based upon shared interests D. Same-grade cohorts avoiding social contact with students from other grades E. Emergence of a charismatic leader who directs the play for the entire group
B Clinical psychiatry Development & Maturation
Which of the following metabolic changes is characteristic of normal aging? A. Insulin levels decline B. Creatinine clearance declines C. Corticotropin levels decrease D. Parathyroid hormone levels decline
B Clinical psychiatry Development & Maturation
A 7-year-old child has an estimated vocabulary of approximately 5000 words, This represents a delay in which of the following domains of language development? A. Grammar B. Semantics C. Pragmatics D. Social communication E. Phonological awareness
B Clinical psychiatry Development & maturation
A sense of idealism and invulnerability that may lead to risky behavior is common in which phase of life? A. Childhood B. Adolescence C. Young adulthood D. Middle adulthood E. Late adulthood
B Clinical psychiatry Development & maturation
A patient is instructed to name the colors in which the series of words are printed, rather than reading the words themselves. This is called the: A Tower Test. B Stroop Test. C Boston Naming Test. D Test of Auditory-Perceptual Skills. E Wisconsin Card Sorting Test.
B Clinical psychiatry Diagnostic procedures
After discontinuing clozapine due to WBC count less than 2,000/mm3 or absolute neutrophil count less than 1,000/mm3, for how long must a patient continue to have a weekly CBC performed once WBC count returns to more than 3,500/mm3 ? A. 1 week B. 4 weeks C. 2 months D. 6 months E. 12 months
B Clinical psychiatry Diagnostic procedures
Which of the following types of electroencephalogram (EEG) monitoring is most likely to aid in differentiating psychogenic from true blindness? A. Sleep deprivation B. Evoked potentials C. 24-hour monitoring D. Polysomnography E. Nasopharyngeal leads
B Clinical psychiatry Diagnostic procedures
Psychological stress increases cortisol synthesis and release, and prolonged, increased cortisol levels have been associated with adverse psychiatric outcomes. Stress also affects the release of other hormones. An increase of which of the following hormones is correlated with decreased posttraumatic stress disorder severity, most likely due to the hormone's anti-glucoconicoid properties? A. Prolactin B. Dehydroepiandrosterone (DHEA) C. Thyroid-stimulating hormone (TSH) D. Corticotropin-releasing hormone (CRH) E. Adrenocorticotropic hormone (ACTH)
B Clinical psychiatry Epidemiology
According to the American Psychiatric Association guidelines, sexual relationships with patients are: A. permitted with former patients only. B. not allowed with current or former patients. C. permitted with parents of child patients, but not with patients themselves. D. permitted when the patient fully understands the implications of the relationship and gives consent. E. not allowed until the psychiatrist terminates the relationship and refers the patient to another provider.
B Clinical psychiatry Issues in Practice
When an incarcerated adult patient is being treated in a correctional setting, a limit to confidentiality that does not exist in other settings is information about: A. communicable diseases such as HIV. B. imminent plans to escape or create disorder. C. threats made against the patient by other inmates. D. illegal activity, but only if it is related to the crime for which the patient is incarcerated.
B Clinical psychiatry Issues in Practice
Eighty percent of the patients in a substance use program are engaged in treatment. Which of the following types of quality measures does this represent? A. Structure B. Process C. Outcome D. Screening E. Satisfaction
B Clinical psychiatry Issues in practice
Which of the following strategies has been most often linked to better outcomes in the treatment of depression in primary care settings? A. Patient education B. The use of care managers C. Continuing medical education D. Published treatment guidelines E. Depression screening measures
B Clinical psychiatry Issues in practice
A 50-year-old patient presents to the emergency department with acute onset of auditory and visual hallucinations. The hallucinations abruptly began several hours prior to presentation. The patient also complains of recently feeling "moody" and "on edge." The patient is alert and fully oriented. Physical examination is notable for tachycardia and diaphoresis. Which of the following disorders does the patient most likely have? A. Opioid use disorder B. Alcohol use disorder C. Brief psychotic disorder D. Bipolar I disorder with psychotic features E. Major depression with psychotic features
B Clinical psychiatry Psychopathology & associated conditions
A patient with a depressive disorder presents with increased appetite and weight gain, hypersomnia, and a heavy feeling in the arms and legs. Which of the following is the most likely mood description? A Irritable B Mood reactivity C Keyed up and tense D Worse in the morning E Elevated and expansive
B Clinical psychiatry Psychopathology & associated conditions
A pt reports a fear of driving and experiences feelings of panic, SOB, palpitations, sweating, clamminess when anticipating a drive down a steep road. The pt acknowledges going out of the way to avoid this situation. They deny similar symptoms in other settings. Which is the most likely diagnosis? A Panic disorder B Specific phobia C GAD D PTSD E OCD
B Clinical psychiatry Psychopathology & associated conditions
A woman presents to the physician for evaluation of inability to achieve orgasm. Presence of which of the following factors would argue against a diagnosis of female orgasmic disorder? A. Patient concerns about her body image B. Intimate partner violence in current relationship C. Religious beliefs with strong prohibitions against sexual activity D. Discrepancy between patient's and partner's desire for sexual activity E. Ability to achieve orgasm during masturbation but not during partnered sexual activity
B Clinical psychiatry Psychopathology & associated conditions
Children with a tic disorder are less likely than adolescents and adults with a tic disorder to experience co-occurrence of: A. specific learning disability. B. major depressive disorder. C. separation anxiety disorder. D. social anxiety disorder (social phobia). E. attention-deficit/hyperactivity disorder.
B Clinical psychiatry Psychopathology & associated conditions
In order to meet criteria A for schizophrenia, a patient must have at least one of three core symptoms: delusions, hallucinations, or: A. negative symptoms. B. disorganized speech. C. marked decrease in functioning. D. abnormal psychomotor behavior. E. grossly disorganized or catatonic behavior.
B Clinical psychiatry Psychopathology & associated conditions
Which of the following most seriously threatens the external validity of a research study? A. Placebo responses in the study population B. A study population that is not representative of the population to be treated C. Mortality or differential attrition during the course of treatment during the study D. Maturation due to natural change during the period between baseline and post-test E. Statistical regression or regression toward the mean
B Clinical psychiatry Research & Scholarship Literacy
A 58-year-old patient with major depressive disorder is undergoing a course of ECT treatment. After the third ECT treatment, the patient develops a dense retrograde amnesia. This side effect can be ameliorated by which of the following? A. Switching from unilateral to bilateral ECT B. Increasing the interval between ECT treatments C. Decreasing the interval of the ECT-induced seizures D. Concurrent administration oftacrine during the course of ECT E. Concurrent administration of methylphenidate during the course of ECT
B Clinical psychiatry Treatment
A patient is prescribed fluoxetine 20 mg daily to treat a first depressive episode. After three weeks the patient reports that the medication is "not working" and continues to endorse mild depressive symptoms. Which of the following interventions is most appropriate? A. Discontinue fluoxetine and start citalopram 20mg per day B. Continue fluoxetine 20mg per day and reassess in two weeks C. Continue fluoxetine 20mg per day and add buspirone 5mg twice daily D. Increase the dose of fluoxetine to 40mg per day E. Discontinue fluoxetine and start bupropion 150mg per day
B Clinical psychiatry Treatment
Biofeedback is a treatment modality used across multiple disorders to help patients achieve greater control over specific physiological functions via which of the following mechanisms? A. Classical conditioning B. Operant conditioning C. Covert sensitization D. Stimulus generalization E. Observational learning
B Clinical psychiatry Treatment
Dialectical behavior therapy is unique among commonly employed psychotherapies in encouraging judicious use by patients of which of the following? A. Keeping a diary B. Telephone consultation C. Brief hospital admissions D. Psychotropic medication E. Consultation with other therapists
B Clinical psychiatry Treatment
Which of the following characteristics of self-help groups distinguishes them from traditional psychotherapy groups? A. Self-help groups focus on modifying personality. B. Goals and objectives do not address intragroup dynamics. C. The leader prohibits extragroup socializing and social contacts. D. Professional involvement of the leader is central to establishing goals. E. Instillation of hope is absent because the specific problem is too far advanced.
B Clinical psychiatry Treatment
Which of the following interventions is an example of a wraparound, in-home, community-based treatment for youth with behavioral disturbances? A. Parent training B. Multisystemic therapy C. Functional family therapy D. Dialectical behavior therapy E. Parent-child interaction therapy
B Clinical psychiatry Treatment
Exposure to light helps entrain the circadian system by activating nonvisual retinal photoreceptors that project to which of the following brain structures? A. Pineal gland B. Suprachiasmatic nucleus C. Tuberomamillary nucleus D. Reticular activating system E. Ventrolateral preoptic nucleus
B Neurosciences
Genetic mapping methods aim to identify disease-associated variants based on which of the following? A. Epigenetics B. Genetic linkage C. Genetic mutation D. Copy number variants E. Phenotypic expression
B Neurosciences
Narcolepsy is believed to be caused by deficiency of which of the following substances? A. Melatonin B. Hypocretin C. Acetylcholine D. Norepinephrine E. Melanin-concentrating hormone
B Neurosciences
Research studies have found an association between Alzheimer disease and the accumulation of which of the following proteins inside conical neurons? A. Amyloid B. Tau C. Fibrillin D. Presenilin E. Neuropeptide Y
B Neurosciences
There is a strong association between HLA B1502 allele and Steven-Johnson syndrome in individuals of Asian descent who are treated with which of the following agents? A. Valproic acid B. Carbamazepine C. Lamotrigine D. Topiramate E. Phenytoin
B Neurosciences
There is robust evidence from epidemiological studies that the offspring of older fathers have an increased risk of which of the following disorders? A. Alcoholism B. Schizophrenia C. Major depression D. Antisocial personality disorder E. Attention-deficit/hyperactivity disorder
B Neurosciences
Which of the following is observed in patients with lesions involving one of the cerebellar hemispheres? A. Gross movements are affected more than fine movements. B. Skilled movements of the extremities ipsilateral to the lesion are disturbed. C. Posture and gait are more severely impaired than for lesions of the vermis. D. Dysmetria and hypotonicity typically affect the leg and foot more than the arm and hand. E. Dysarthria and articulation disturbances are often present and difficult to discriminate from a stroke.
B Neurosciences
Which of the following is the causative agent of progressive multifocal leukoencephalopathy? A Prions B. JC virus C. Cytomegalovirus D. Herpes simplex virus type I E. Human herpes virus type 6
B Progressive multifocal leukoencephalopathy (PML) is a disease of the white matter of the brain, caused by th JC virus infection that targets cells that make myelin--the material that insulates nerve cells (neurons). Clinical neurology
An 87-year-old patient who is a college graduate with longstanding cardiovascular disease and dyslipidemia is brought to the psychiatrist by an adult daughter who lives with the patient and has become concerned about the patient's forgetfulness and confusion. The patient receives a score of 21/30 on the Folstein Mini-Mental State Examination (MMSE). This score is consistent with: A. normal cognitive decline. B. mild cognitive impairment. C. moderate cognitive impairment. D. severe cognitive impairment. E. probable sensory impairment.
B (I disagree with this) Clinical psychiatry Diagnostic procedures
A patient with Alzheimer disease has been treated with donepezil for the past few years. The dementia has progressed, and the patient now presents with significant agitation, lability, and appetite disruption. A medical workup is unrevealing. Which of the following would be the most appropriate medication to add to donepezil? A. Citicoline B. Memantine C. Galantamine D. Rivastigmine E. Physostigmine
B Memantine with Donepezil Memantine and donepezil combination is used to treat dementia (memory loss and mental changes) associated with moderate or severe Alzheimer's disease. Clinical psychiatry Treatment
Most patients presenting for detoxification from alcohol are treated on an outpatient basis, Which of the following risk factors would lead most clinicians to recommend inpatient treatment for a new patient? A. Age under 50 years B. History of a prior withdrawal seizure C. First-degree relative with delirium tremens D. Hepatic enzymes elevated twice the upper limit of normal E. History of multiple arrests for driving under the influence
B previous w/d seixure Clinical psychiatry Epidemiology
A consultation-liaison psychiatrist evaluates a 40-year-old patient who has had an uncomplicated hospital course following surgery under general anesthesia. On post-op day four the patient develops new-onset hallucinations. Which of the following is the most likely cause? A. Schizophrenia B. Delirium tremens C. Acute stress disorder D. Brief psychotic disorder E. Major depression with psychotic features
B. Delirium tremens Clinical psychiatry Consultation/collaborative innovative care
Which of the following is the infectious neuropsychiatric disorder characterized by psychosis and abnormal gait, that was once treated with malaria-induced fever and accounted for up to 20% of patients in mental asylums until the 1940s? A. Polio B. Syphilis C. Lyme disease D. Herpes simplex virus E. Post-streptococcal infection
B. Syphilis Clinical psychiatry Issues in practice
Interpersonal psychotherapy of adults with depression begins with assessment of the patient's adjustment to stressful life events, role transitions and disputes, and: A. ability to learn from interpretation. B. quality of social connections. C. early childhood experiences. D, unconscious conflicts. E. character traits.
B. quality of social connections. Clinical psychiatry Treatment
- BMD is an X-linked recessive muscular dystrophy that results in slowly progressing muscular weakness in the legs and pelvis. Compared to DMD
BMD symptoms occur later in life, often in the pre-teen years.
- Episodic vertigo and is brought on by head movements or changes in posture and often associated with nausea and nystagmus- -What is the Dx? -Testing: -Treatment?
BPPV is due to abnormal stimulation of the semicircular canals by otoliths from the utricle. Diagnosis may be aided by provoking nystagmus and vertigo with the Dix-Hallpike maneuver and otolith replacement maneuvers such as the Epley maneuver may improve symptoms. Vertigo involves a sensation of movement (either the environment or the individual), and is important to distinguish from other common causes of "dizziness," including light-headedness (presyncope or hypotension) or instability (ataxia or other gait disorder). Meniere's disease may also result in episodic vertigo, but is typically associated with tinnitus, sensation of ear fullness, and low-frequency hearing loss.
- Patient with HLA- B*1502 allele who takes Carbamazepine is at risk for?
Black box warning for severe dermatologic reactions and aplastic anemia and agranulocytosis.
A patient presents for an evaluation of "memory problems. The patient reports frequent episodes of ending up somewhere without any memory of getting there. The patient also occasionally forgets how to perform job functions, and has gaps in memories of high school. In addition, the patient endorses the occasional experience of observing the self from outside the body, and notes periods of being suddenly overcome by unexpected emotions. Recently the patient began hearing a child's voice. Which of the following is the most likely diagnosis? A. Malingering B. Dissociative amnesia C. Dissociative identity disorder D. Borderline personality disorder E. Depersonalization/derealization disorder
C presence of 2 or more distinct identities or personality states that recurrently take control of behavior, more common in women -associated with a history of sexual abuse -loss of memory for traumatic or every day activities Treatment psychotherapy Clinical psychiatry Psychopathology & associated conditions
Clear evidence for adult neurogenesis has been established in which of the following areas of the brain? A. Amygdala B. Cerebellum C. Hippocampus D. Olfactory bulb E. Anterior cingulate cortex
C "Hippos and Campus' keep getting bigger" Neurosciences
A patient with autism has had significant difficulties with disruptive behavior, loud outbursts, and functional difficulties over the past year. The patient has been actively engaged in outpatient treatment without significant progress despite intensive psychotherapy and psychopharmacologic treatment. There is no evidence of suicidality or aggression. Which of the following is the most appropriate treatment step for this patient? A. Boarding school B. Residential treatment C. Partial hospitalization program D. Inpatient admission for acute stabilization E. Continue outpatient treatment with closer follow-up
C ABA, partial hospitalization, escalate up steps. Clinical psychiatry Treatment
The presence of which of the following would help in determining if a tonic-clonic seizure-like episode was non-epileptic rather than an epileptic seizure? A. Self-injury B. Stereotyped episodes C. Lack of elevation of prolactin D. Rhythmic movements of arms E. Presence of postictal confusion
C Clinical Neurology
A 62-year-old patient with poor nutrition complains of difficulty walking. Examination is remarkable for hyperreflexia with marked reduction in vibration and position sense at the toes. A diagnosis of cobalamin (vitamin 1312) deficiency is suspected, but the results of cobalamin (vitamin B12) testing reveal a borderline low value. Which of the following lab tests would confirm the suspected diagnosis? A. Folate B. Glutathione C. Methylmalonic acid D. Pyridoxine (vitamin B6) E. Erythrocyte sedimentation rate
C Clinical neurology
A 70-year-old patient develops confusion, lethargy, and a generalized tonic-clonic seizure. Laboratory testing reveals a serum sodium of 95 mEq/l. Which of the following is the most likely complication of excessively rapid correction of this metabolic problem? A. Basilar artery stroke B. Dialysis encephalopathy C. Central pontine myelinolysis D. Portal-systemic encephalopathy E. Non-convulsive status epilepticus\
C Clinical neurology
An 80-year-old patient is evaluated for visual hallucinations and worsening gait. The patient's spouse reports that the patient has had episodic confusion and disturbed sleep in which the patient shouts out and appears to be fighting someone. Previous trials of levodopa/carbidopa have resulted in temporary improvement in motor function. Examination is remarkable for bilateral rigidity, masked facies and gait instability. Which of the following is the most likely diagnosis? A. Alzheimer disease B. Multisystem atrophy C. Dementia with Lewy bodies D. Idiopathic Parkinson disease E. Corticobasal ganglionic degeneration
C Clinical neurology
Which of the following is the most effective treatment of focal hand dystonia associated with writing ("writer's cramp")? A. Carbamazepine B. Valproic acid C. Botulinum toxin D. Glatiramer acetate E. Levodopa/carbidopa
C Clinical neurology
Which of the following types of medication can precipitate REM sleep behavior disorder? A. Stimulant B. Benzodiazepine C. Selective serotonin reuptake inhibitor D. Typical (first generation) antipsychotic E. Atypical (second generation) antipsychotic
C Clinical neurology
Proponents of which of the following theories agree that social norms, expectations, meaning and perceptions are keys to understanding individual behavior in institutions and institutional effectiveness? A. Open systems B. Human relations C. Organizational culture D. Structural and sociotechnical E. Organizational power and conflict
C Clinical psychiatry Administrations & systems
A bookkeeper who is embezzling money justifies this behavior by noting that the company is still making a profit. This scenario exemplifies which of the following defense mechanisms? A. Idealization B. Sublimation C. Rationalization D. Intellectualization E. Projective identification
C Clinical psychiatry Behavioral & social sciences
An individualized education program (IEP) offers a wider range of options than a 504 plan for schools to meet children's educational needs because an IEP: A. has a lower standard for eligibility. B. allows accommodations for psychiatric illness. C. allows modification to the academic requirements for the student. D. allows schools to discipline any disruptive behavior as they see fit. E. allows schools to evaluate family members of the student if necessary.
C Clinical psychiatry Consultation/collaborative integrated care
In a community mental health center providing recovery-oriented care, which aspect of the psychiatrists' role is based on recovery-oriented principles? A. Providing trauma-informed care B. Leadership of the multidisciplinary team C. Collaborating in partnership with consumers to support their goals D. Consultation to and supervision of other mental health professions E. Providing diagnostic evaluation and medication management of mental disorders
C Clinical psychiatry Consultation/collaborative integrated care
When treating psychiatrists are asked by the judicial system to testify on behalf of their patients, ethical considerations, the doctor-patient relationship, and legal guidelines indicate that they can comment on which of the following topics? A Damages B Impairment ratings C Clinical presentation D Causation of injury E Extent of disability
C Clinical psychiatry Consultation/collaborative integrated care
A 38-year-old single woman presents to the psychiatrist reporting difficulty dealing with the prospect of not having children. She has always wanted to be a mother, and given her age and lack of current romantic involvement, she fears that motherhood may not be possible. She becomes a little tearful as she discusses this, and wonders whether she should consider adoption as a single mother. Which of the following is the most likely diagnosis? A. Relational problem B. Adjustment disorder C. Phase of life problem D. Generalized anxiety disorder E. Unspecified depressive disorder
C Clinical psychiatry Development & Maturation
The adult child of a 75-year-old parent is concerned that the parent is no longer safe to drive, after getting into several recent car accidents. The parent is in good physical health, has no chronic diseases, and is on no medications. Which of the following is most likely the cause of the motor vehicle accidents? A. Hearing loss B. Hand tremors C. Slower visual processing D. Impaired word processing E. Decreased muscle strength
C Clinical psychiatry Development & Maturation
A child spends several minutes talking with parents recounting how a peer "got in trouble today." The child continues in great detail about how the peer is always breaking the rules. This child is most likely in which of the following age groups? A. Preschool B. Toddlerhood C. School-age D. Early adolescence E. Late adolescence
C Clinical psychiatry Development & maturation
Which of the following characteristics would be most expected of infants who are hypersensitive and less able to tolerate environmental stimuli? A. Lethargic and often withdrawn B. Seldom distressed and often pensive C. Easily startled and more frequent crying D. Recognizing and giving less attention to the familiar E. Inability to discriminate mother's voice from a stranger's
C Clinical psychiatry Development & maturation
Which of the following interview techniques is being used when asking the patient, "What brought you to the hospital today?" A Confrontation B Attentive listening C Open-ended questioning D Maintenance of neutrality E Closed-ended questioning
C Clinical psychiatry Diagnostic procedures
A media campaign is launched to reduce the incidence of suicides by military veterans. The campaign includes television, radio, and print advertisements that provide information about warning signs, directions on how to get help, and evidence about the effectiveness of treatment. This is an example of which of the following public health strategies? A. Secondary prevention B. Teniary prevention C. Universal intervention D. Selective intervention E. Indicated intervention
C Clinical psychiatry Epidemiology
A patient with moderately severe dementia participates in a randomized controlled trial of a new cognitive enhancer. Proxy consent was obtained from the patient's spouse, After 3 weeks, the patient tells the psychiatrist he no longer wishes to participate in the study because of nausea he attributes to the drug but the spouse wants the patient to continue taking it. Which of the following should the psychiatrist do? A. Continue the patient in the study as it is in his best interest B. Continue the patient in the study as the spouse has provided proxy consent C. Withdraw the patient from the study because not doing so breaches his autonomy D. Withdraw the patient from the study because not doing so may cause significant harm
C Clinical psychiatry Issues in Practice
A 17-year-old patient is brought by family to the emergency department due to severe symptoms of restlessness, nervousness, facial flushing, muscle twitching, and tachycardia. Family became most concerned when the patient began to have rambling flow of thought and speech, and was becoming agitated. The patient has most likely been drinking a large quantity of: A ginger ale. B. sarsaparilla. C. energy drinks. D. ginseng soda. E. pomegranate juice.
C Clinical psychiatry Psychopathology & associated conditions
A male physician is repeatedly accused by adult female patients of sitting opposite them on his stool during an eye examination, opposing knees alternating, in such a manner that one of the patients' knees touches the physician's crotch, If true, this is an example of: A. fetishism. B. partialism. C. frotteurism. D. sexual sadism. E. sexual masochism.
C Clinical psychiatry Psychopathology & associated conditions
A patient consults the psychiatrist for nervousness felt in many public situations. The patient, who has a very mild stutter, fears being viewed by others as stupid and inept. The patient reports being in a very stable marriage with mutually satisfying intimacy but was recently passed over for a promotion at work due to "not being a team player. Most of the patient's meals are eaten at home, as dining in restaurants and other public places elicits discomfort. Which of the following is most likely? A. Agoraphobia B. Normal shyness C. Social anxiety disorder D. Avoidant personality disorder E. Anxiety related to a medical condition
C Clinical psychiatry Psychopathology & associated conditions
Hoarding disorder is most commonly accompanied by which of the following disorders? A. Bipolar B. Agoraphobia C. Major depressive D. Obsessive-compulsive E. Attention-deficit/hyperactivity
C Clinical psychiatry Psychopathology & associated conditions
When evaluating a patient in the emergency department, which of the following symptoms most strongly suggests a general medical condition? A Anergia B Irritability C Clouded sensorium D Psychomotor agitation E Auditory hallucinations
C Clinical psychiatry Psychopathology & associated conditions
Which of the following is a predisposing vulnerability factor in posttraumatic stress disorder (PTSD)? A. Male gender B. Schizotypal traits C. Recent excessive use of alcohol D. Perception of internal locus of control E. Obsessive-compulsive personality traits
C Clinical psychiatry Psychopathology & associated conditions
Which of the following is characteristic of persons diagnosed with rapid eye movement(REM) sleep behavior disorder? A. Disorder overwhelmingly affects females B. Episodes are commonly associated with daytime naps C. Polysomnography is notable for REM sleep without atonia D. Episodes occur within the first 60 minutes after sleep onset E. Upon waking from an episode, these individuals are usually confused and disoriented
C Clinical psychiatry Psychopathology & associated conditions
Which of the following is the most common type of dissociative amnesia? A. Failure to recall new events as they occur B. Total loss of memory for one's own personal history C. Failure to recall events during a particular period of time D. Inability to recall some, but not all, events during a particular time E. Inability to recall information in a particular category (e.g., related to one's family)
C Clinical psychiatry Psychopathology & associated conditions
Which of the following psychotherapy modalities is most commonly used in crisis management? A. Psychoanalysis B. Behavioral therapy C. Supportive therapy D. Interpersonal psychotherapy E. Psychodynamic psychotherapy
C Clinical psychiatry Psychopathology & associated conditions
A researcher Investigates whether heavy cannabis use in adolescence is associated with schizophrenia. Daily cannabis use during adolescence in 200 adult participants with schizophrenia is compared with use in 200 participants without a psychiatric diagnosis, Which of the following study designs is the researcher using? A. Cohort study B. Ecological study C. Case-control study D. Cross-sectional study E. Randomized control trial
C Clinical psychiatry Research & scholarship literacy
A married couple is seeking therapy to help with issues of mutual resentment. The wife has already been in individual therapy for several months and has a high regard for her therapist. She asks if her therapist can conduct the couples therapy as well. Which of the following is the therapist's best response? A. "I would be happy to. I really think your husband needs to hear what you been going through. " B. "No, it would be better for your husband's therapist to do the therapy, as he can represent your husband's point of view. " C. "A couples therapist needs to be impartial, and I already have a relationship with you. " D. "I'd enjoy that, but I can't fit you into my schedule right now; let me suggest a colleague.
C Clinical psychiatry Treatment
A patient is referred to psychiatry by the primary care provider (PCP). The PCP informs the psychiatrist that the patient repeatedly complains of abdominal pain and nausea, but an extensive medical work-up was within normal limits. The patient recently immigrated to the United States, and the PCP wonders whether the patient's presentation is a culture-bound syndrome. Asking the patient which of the following questions is most likely to yield information that helps with clinical formulation? A. "Are you homesick?" B. "Why don't you believe the test results?" C. "What do you think is causing your problem?" D, "Do you have trouble expressing your feelings?" E. "Is there something that you are not telling your doctor?"
C Clinical psychiatry Treatment
A patient with traumatic brain injury is trained to use an electronic tablet to keep track of appointments and other details of life, and works monthly with a therapist to help successfully integrate the use of it into everyday life. Which of the following interventions best describes this type of intervention? A. Milieu therapy B. Harm reduction C. Cognitive rehabilitation D. Motivational interviewing E. Supportive psychotherapy
C Clinical psychiatry Treatment
An interpersonal psychodynamic group comprised of people with heterogeneous diagnoses and levels of functioning has room to take a new member. When considering patients to refer to the group, which of the following diagnoses would be a relative contraindication? A. Bipolar disorder B. Major depression C. Antisocial personality disorder D. Dependent personality disorder E. Narcissistic personality disorder
C Clinical psychiatry Treatment
Co-prescription of which of the following analgesic medications is likely to cause a clinically significant rise in lithium level? A. Acetaminophen B Amitriptyline C. Celecoxib D. Duloxetine E. Oxycodone
C Clinical psychiatry Treatment
A patient with Parkinson disease develops a change in behavior characterized by frequent, compulsive gambling. Which of the following medications is most likely contributing to this change in the patient's behavior? A. Levodopa B. Selegiline C. Pramipexole D. Benztropine E. Entacapone
C Dopamine agonists for parkinsons and restless legs, Bromocriptine (Parlodel), Cabergoline, Apomorphine (Apokyn), Pramipexole (Mirapex), Ropinirole (Requip), Rotigotine (Neupro). Also some case reports of Abilify: Clinical neurology
Hyperprolactinemia with antipsychotic medication use increases risk of which of the following? A. Seizures B. Hypersexuality C. Osteoporosis D. Hallucinations E. Heart disease
C Neurosceinces
Conical columns are vertical collections of neurons whose role is best understood as: A. signaling units for critical periods. B. regulatory units for long-term potentiation. C. functional units for information processing. D. redundant backups for subcortical processing. E. reserve units underlying experience-dependent plasticity.
C Neurosciences
Increased risk-taking behavior is most likely to occur as a result of a lesion in which of the following brain regions? A. Precuneus B. Cerebellar vermis C. Orbital frontal cortex D. Medial temporal cortex E. Parahippocampal gyrus
C Neurosciences
Pain caused by neurogenic inflammation is mediated by which of the following? A. Metalloproteinase B. Interleukin 6 C. Substance P D. Dopamine E. Complement
C Neurosciences
The 5-HT3 receptor is best described as a(n): A. intra-cellular receptor. B. mitochondrial receptor C. ligand gated ion channel. D. voltage gated ion channel. E. G-protein coupled receptor.
C Neurosciences
The rate of synaptogenesis peaks at roughly which of the following times in human development? A. Second trimester of pregnancy B. Third trimester of pregnancy C. The first four years of life D. Between 5 and 10 years of life E. Between 15 and 20 years of life
C Neurosciences
Which of the following methods is used to confirm the diagnosis of velocardiofacial syndrome (22q.11 microdeletion)? A. Karyotyping B. Genetic linkage analysis C. Fluorescent in situ hybridization D. Trinucleotide expansion analysis E. Mitochondrial DNA deletion analysis
C Neurosciences
Which of the following would be most useful for enhancing the doctor-patient relationship and improving emotional communication when using telemedicine to interview a patient? A. Increasing the volume of the audio to near maximum volume B. Recording the interchange so that the patient can review it later C. Positioning the webcam close to the image of the patient on the screen D. Using a wide zoom so that the psychiatrist can see the patient's entire body E. Using shorter sentences with more frequent pauses than would be done in person
C Positioning the webcam close to the image of the patient on the screen Clinical psychiatry Issues in practice
Which of the following physiologic changes best explains why older adults, especially people over age 70, are at increased risk of developing extrapyramidal symptoms during antipsychotic medication treatment? Decreased: A. monoamine oxidase activity B. acetylcholinesterase activity C. number of available dopamine receptors D. renal clearance of antipsychotic medications E. volume of distribution of antipsychotic medications
C Studies show that D2 dopamine receptor binding sites (mainly postsynaptically located) decrease as a consequence of normal aging in parallel with the decline of the presynaptic nigrostriatal dopaminergic neuronal system. Clinical psychiatry- Treatment
The case of "Little Hans" led Freud to develop a psychological theory of the formation of which of the following symptoms? A. Depression B. Regression C. Phobia D. Inattention E. Mania
C Herbert Graff- a friend's son, "Little Hans", was the subject of Freud's early but extensive study of castration anxiety and the Oedipus complex. Was known to have a fear of horses. Clinical psychiatry- Issues in practice
A 70-year-old man presents with a 4-month history of worsening cognitive decline, urinary incontinence and difficulty walking. Which of the following are the most likely findings on this patient's structural magnetic resonance imaging (MRI) scan? A. Cortical atrophy B. Cerebellar atrophy C. Enlarged ventricles D. Striatal degeneration E. Diffuse small cortical infarcts
C Normal Pressure Hydrocephalus- wide based magnetic gait, urinary incontinence, Next step: Large Volume LP -gait improves Tx: ventricular shunting Clinical psychiatry Diagnostic procedures
Prostaglandin D2 helps regulate the sleep-wake cycle by increasing extracellular levels of which of the following chemicals? A. GABA B. Hypocretin C. Adenosine D. Melatonin E. Histamine
C- PG D2 increases Adenosine to induce sleep Hypocretin also known as orexin) is a neuropeptide hormone produced in the hypothalamus that exerts important influences over sleep, arousal, appetite and energy expenditure. Defective hypocretin signaling is associated with narcolepsy. Melatonin is a hormone primarily released by the pineal gland at night to support sleep Neurosciences
Which of the following is characteristic of acetylcholine receptors found in the cerebral cortex? A. Blocked by curare B. Predominantly nicotinic C. Both inhibitory and excitatory D. Increased in Alzheimer disease E. Predominantly found in the substantia nigra
C. Both inhibitory and excitatory - Neurosciences
Which of the following is the strongest predictor of engagement in leadership roles by patients in mental health consumer-run organizations? A. Race or ethnicity of patient B. Availability of peer counseling C. Perceived sense of community D. Perceived organizational empowerment E. Number of self-help group meetings offered
C. Perceived sense of community Clinical psychiatry Administration & systems
The purpose of adding oral naloxone to buprenorphine for opioid replacement therapy is because its opioid antagonism prevents which of the following? A. Euphoria from the buprenorphine B. Overdose if extra doses are ingested C. Intravenous abuse of the buprenorphine D. Euphoria if additional types of opioids are used E. Overdose if additional types of opioids are used
C= prevent buprenophine Clinical psychiatry Treatment
citalopram, escitalopram and amitriptyline are metabolized by which CYP?
CYP2C19 mainly metabolizes the antidepressants citalopram, escitalopram and amitriptyline17 and contributes to the metabolism of sertraline.
- Fluvoxamine an inhibitor of CYP 3A3 and 3A4, 2C9, 2C19, 1A2 and of 2D6 at high doses, however it interacts with caffeine via CYP 1A2. CYP1A2 inhibition is the correct answer option.
Caffeine clearance is lowered by 80% when consumed with fluvoxamine, resulting in elevated caffeine levels.
What mood stabilizer is best for breastfeeding moms?
Carbamazepine can be given to breastfeeding mothers as very low serum levels are found in breast milk so this is a viable option.
- All of the cholinesterase inhibitors have been found to be equally effective in the treatment of NCD due to Alzheimer's disease.
Cholinesterase inhibitors such as donepezil, galantamine, Rivastygmine
What medications used for Alzheimer's disease may cause Bronchoconstriction in asthma and COPD is mediated by the vagus nerve by cholinergic neurons through the parasympathetic nervous system. Patients with a history of asthma or COPD potentially can experience increased airway reactivity following cholinergic stimulation?
Cholinesterase inhibitors such as donepezil, galantamine, Rivastygmine used in the treatment of Alzheimer's disease.
- Oral contraceptive can do what to caffeine?
Contraceptives inCrease Caffeine and can predispose to caffeine intoxication.
CYP PROFILES
Cytochromes P450 (CYPs) are a superfamily of enzymes containing heme as a cofactor that functions as monooxygenases. These proteins oxidize steroids, fatty acids, and xenobiotics, and are important for the clearance of various compounds, as well as for hormone synthesis and breakdown.
Which of the following developmental syndromes results from a deletion that spans a number of genes? A. Fragile X B. Klinefelter C. Turner D. Williams E. Prader-Wi11i
D "D. Williams- is Deleted from NBA" Neurosciences
A 68-year-old patient recently had a stroke and now exhibits neglect of the left side. What region of the brain was most likely affected? A Left frontal lobe B Right frontal lobe C Left parietal lobe D Right parietal lobe E Right temporal lobe
D "Right Parietal- Left Neglect" Neuroscience
A psychiatrist treats a patient in psychotherapy for depression and anxiety. The patient's partner has received a prestigious job offer in another city. Although the patient can easily find work in the other city, the patient feels angry about the possibility of having to move and is thinking of ending the relationship. The patient also reveals feeling a loss of control. According to Erik Erikson, this patient is struggling to navigate which stage of psychosocial development? A. Autonomy versus shame and doubt B. Identity versus role confusion C. Generativity versus stagnation D. Intimacy versus isolation E. Integrity versus despair
D 1Trust vs. mistrust Birth to 12-18 mo A sense of trust and security 2Autonomy vs. shame & doubt 18 mo- 3 yr Feelings of independence lead to belief in yourself and your abilities 3 Initiative vs. guilt 3 to 5 years Self-confidence; the ability to take the initiative and make decisions 4 Industry vs. inferiority 5 to 12 years Feelings of pride and accomplishment 5 Identity vs. confusion 12 to 18 years A strong sense of identity; a clear picture of your future 6 Intimacy vs. isolation 18 to 40 years Safe relationships filled with commitment and love 7 Generativity vs. stagnation 40 to 65 years The desire to give to family and community, and to succeed at work 8 Integrity vs. despair Over 65 years Pride in what you've achieved leads to feelings of satisfaction Clinical psychiatry Development & maturation
138. Abortive treatment of common migraine is best achieved with which of the following medications? A. Duloxetine B. Topiramate C. Desipramine D. Rizatriptan
D Clinical neurology
A 28-year-old woman is hospitalized with symptoms of paranoid delusions, auditory hallucinations, and agitation. When treated with haloperidol 5 mgs the patient is noted to become rigid and mute. Within days she develops choreiform movements, has a seizure and develops respiratory difficulty leading to an ICU admission. An examination of the patient, which includes an abdominal ultrasound, reveals an ovarian mass. A lumbar puncture is performed, Cerebrospinal fluid (CSF) is likely to show antibodies to which of the following? A. VGKC-complex B. AMPA receptor C. GABA B receptor D. NMDA receptor E. Glycine receptor
D Clinical neurology
A 63-year-old patient with a history of excess alcohol intake presents with confusion. The patient is unaware of surroundings and answers only some questions with brief responses. On examination the patient is unable to move eyes laterally on either side and has nystagmus when attempting lateral gaze. The patient is unable to stand without assistance. The patient's symptoms are most likely a result of deficiency of: A. iron. B. folate. C. glucose. D. thiamine. E. vitamin D.
D Clinical neurology
Global mental health most differs from other areas of global medicine in which of the following ways? A. The effects of mental health treatment are simpler to quantify. B. Psychiatry places a greater emphasis on cure rather than care. C. The treatments required can usually occur in a briefer period of time. D. Cultural beliefs have a greater influence on diagnostic understandings. E. There is less demand for psychiatric services than other types of medical care.
D Clinical psychiatry Administration & systems
Studies of education campaigns aimed at reducing the stigma of mental illness have reached the unexpected finding that: A. being given a treatment for a mental illness, even if it is effective, increases stigma. B. the public is more accepting of mental illnesses that are considered to be unpredictable. C. stigma around mental illness is not related to the public perception of dangerousness. D. biological explanations for mental illness are not associated with more tolerant attitudes to the illness.
D Clinical psychiatry Behavioral & Social Sciences
A patient has the goal of cleaning out the garage, but becomes extremely anxious and overwhelmed by the thought of even starting to work on it. The patient's therapist suggests breaking the job into manageable pieces to make it seem less overwhelming. Which of the following cognitive-behavioral techniques is this describing? A. Activity scheduling B. Relaxation training C. Behavioral activation D. Graded task assignments E. Exposure and response prevention
D Clinical psychiatry Behavioral & social sciences
The clinical observation that depressed adolescents have difficulty with flexible problem-solving has led to studies demonstrating deficits in tests such as the Trails Making Test B. If these findings are sufficiently replicated it would suggest that, for adolescents, a major depressive disorder causes deficits in which of the following domains of cognitive functioning? A Verbal memory B. Working memory C. Processing speed D. Executive functioning E. Visual-spatial memory
D Clinical psychiatry Behavioral & social sciences
Which of the following is a premise of the objects relations theory? A Everyone develops true and false selves B Aggression and envy are central to psychopathology C The "paranoid position" is representative of the first 6 months of life D The mind is inhabited by mental representations of the self and others E The pleasure principle is central to understanding unconscious conflicts
D Clinical psychiatry Behavioral & social sciences Object relations- is a variation of psychoanalytic theory that diverges from Sigmund Freud's belief that humans are motivated by sexual and aggressive drives, suggesting instead that humans are primarily motivated by the need for contact with others—the need to form relationship with the mother. Melanie Klein's work tends to be most commonly identified with the terms "object relations theory". It maintains that the infant's relationship with the mother primarily determines the formation of its personality in adult life.[2] Particularly, the need for attachment is the bedrock of the development of the self or the psychic organization that creates the sense of identity. object relations theory, the way mothers and infants interact plays a crucial role in infant growth and development. If care is adequate or "good enough," children are able to develop their true selves, which is the part of the baby that is creative and spontaneous. ORT Defense Mechanisms 1) Introspection 2) Projection- discard good/bad to alleviate anxiety 3) Spitting- infant forms good/bad self images to better control the destructive and pleasurable impulses. 4) Projective identification- separating the bad aspects from one self and projecting them into an external object. This bad aspect is distorted and in turn is introjected back into the infant. This process is in place to allow the person to gain control over both the good and bad sides of an object.
The parents of a four-and-a-half-year-old child report that the child insists there is a small monkey who shares the child's room. The child talks to the monkey, asks others to converse with the monkey and insists that the monkey have a seat at the dinner table. The parents ask if they should be concerned. The best response to the parents is that their child is exhibiting: A. early signs of psychosis. B. signs of visual abnormalities. C. signs of exposure to trauma. D. behavior typical for preschoolers. E. early signs of developmental delay.
D Clinical psychiatry Development & Maturation
Which of the following is the most common fear expressed by adults leaving home to enter hospice? A. Pain B. After-life C. Losing control D. Separating from loved ones E. Becoming a burden on others
D Clinical psychiatry Development & Maturation
According to the 2013 Youth Risk Behavior Surveillance Survey which of the following is a recent trend in adolescent sexuality? A. Most adolescents experience sexual intercourse. B. Adolescents' first sexual experience is almost never with a steady relationship partner. C. The proportion of sexually experienced adolescents has increased since the mid-1980's. D. Early age at first sexual intercourse is associated with having more sexual partners. E. About half of never-married adolescents have had six or more partners since their sexual debut.
D Clinical psychiatry Development & maturation
The primary psychological task of school-age children is achieving which of the following? A. Identity B. Autonomy C. Attachment D. Competence E. Individuation
D Clinical psychiatry Development & maturation
Which of the following statements is accurate about sexuality and aging? A. Older women are more sexually active than older men. B. Past levels of sexual activity do not predict later levels of activity. C. Fewer than half of older adults express satisfaction with their sex lives. D. A majority of middle-aged and older individuals continue to be sexually active. E. Physical health is the most important factor determining sexual activity for both sexes.
D Clinical psychiatry Development & maturation
The psychiatrist evaluates a patient whose spouse describes the patient's personality change over the past year including shoplifting, lying, flirtatious behavior, craving foods with sugar, and hoarding canned goods. The patient has no past psychiatric history, and scores 20/30 on the Montreal Cognitive Assessment (MoCA). Which of the following would be the most useful in supporting a diagnosis? A. Electroencephalogram (EEG) B. Diffusion tensor imaging (DTI) C. Dopamine transporter (DaT) scan D. Single-photon emission computed tomography (SPECT)
D Clinical psychiatry Diagnostic procedures
102. Which of the following tools can be used when a person wishes to assign a surrogate to make health care decisions during future periods of mental incapacity? A. Guardianship B. Conservatorship C. Guardian ad litem D. Advance directive E. Representative payee
D Clinical psychiatry Issues in practice
A psychiatrist has recently graduated from residency training and is setting up a private practice. The psychiatrist is personally an avid user of social media and wants patients to be able to access the office through social media. The psychiatrist should: A. send "friend" requests to all new patients. B. not send "friend" requests to patients, but accept them from patients. C. not use social media for professional purposes as it violates patient privacy. D. set up a professional account completely separate from any personal accounts. E. not use social media professionally, but use email to communicate with patients and potential patients.
D Clinical psychiatry Issues in practice
Among individuals with anorexia nervosa, those with binging-purging type are more likely than those with restrictive type to present with comorbid: A. anxiety. B. depression. C. malnutrition. D. substance use.
D Clinical psychiatry Psychopathology & Associated Conditions
Withdrawal from which of the following substances can constitute a life-threatening medical emergency? A. Cocaine B. Opioids C. Phencyclidine (PCP) D. Sedative-hypnotics E. MDMA (ecstasy)
D Clinical psychiatry Psychopathology & Associated Conditions
An 8-year-old child with mild intellectual disability is most likely to have difficulty with which of the following? A. Naming colors B. Following the bedtime routine C. Brushing teeth without supervision D. Understanding social cues of peers E. Accepting hugs from family members
D Clinical psychiatry Psychopathology & associated conditions
For a person to meet diagnostic criteria for enuresis, the behavior must be: A. involuntary in nature. B. during nighttime sleep. C. attributable to a medical condition. D. in a child of at least 5 years of age. E. associated with the effects of a substance.
D Clinical psychiatry Psychopathology & associated conditions
Which of the following is the most clinically significant critique of the use of randomized controlled studies as the gold standard level of evidence? A. They are impractical, costly, and too difficult to perform in most situations. B. They tend to investigate hypotheses that can be more easily answered by other study designs. C. True randomization is a mathematical impossibility, thus group assignments are always biased. D. Their design is better suited to provide information about efficacy rather than effectiveness. E. They are more useful for investigating negative (i.e., adverse) effects than positive outcomes,
D Clinical psychiatry Research & scholarly literacy
The reliability of an assessment instrument, such as a diagnostic interview, refers to the ability of an instrument to: A measure what it intends to measure. B yield low false-positive rates of mistakenly identifying subjects. C. yield adequate information without being unduly time consuming. D. yield consistent results when used by different examiners or at different times. E. detect true cases of particular disorders, thus yielding low false-negative rates.
D Clinical psychiatry Research & scholarship literacy
Which of the following is the main advantage of parallel group, randomized controlled designs compared to crossover designs? A. Less expensive B. More time required C. Increased precision D. Fewer residual effects E. Fewer subjects needed
D Clinical psychiatry Research & scholarship literacy
A patient with bipolar disorder has been clinically stable on Lamotrigine 200 mg daily for the past 6 months. The patient contacts the psychiatrist to request a refill of Iamotrigine, explaining that, "I ran out five days ago but kept forgetting to call." The patient denies any mood symptoms or physical complaints. The most appropriate next step is to restart the lamotrigine at: A. full dose (200 mg daily). B. half the prior dose (100 mg daily). C. 25 mg daily and titrate to 200 mg daily over I week. D. 25 mg daily and follow standard initial titration schedule.
D Clinical psychiatry Treatment
The main goal of group cognitive behavior therapy for chronic pain treatment is to: A. decrease individual members' pain. B. convince family members that chronic pain is real. C. utilize the group process to avoid thinking about stress. D. improve patients' functioning despite the presence of pain. E. reinforce distorted automatic thoughts about the pain experience,
D Clinical psychiatry Treatment
Which of the following SSRIs has the greatest potential for discontinuation syndrome with missed doses? A. Fluoxetine B. Sertraline C. Citalopram D. Paroxetine E. Escitalopram
D Clinical psychiatry Treatment
Which of the following dietary supplements has demonstrated some efficacy in the treatment of depression? A. Glycine B. Melatonin C. Phosphatidylserine D. S-adenosyl-l-methionine (SAMe) E. Nicotinamide adenine dinucleotide (NADH)
D Clinical psychiatry Treatment
Which of the following is the first step in the traditional twelve-step program of addiction recovery? A. Confessing mistakes B. Having faith in a greater power C. Taking a moral inventory of oneself D. Admitting powerlessness over the addiction E. Making amends with those one has harmed
D Clinical psychiatry Treatment
Which of the following medications decreases the efficacy of oral contraceptives? A. Bupropion B. Lithium C. Lorazepam D. Oxcarbazepine E. Fluoxetine
D Clinical psychiatry Treatment
Which of the following psychiatric medications should be discontinued prior to ECT? A. Olanzapine B. Fluoxetine C. Memantine D. Lorazepam E. Zolpidem
D Clinical psychiatry Treatment
36. In the early stages, Huntington disease is characterized by loss of neurons that produce which of the following neurotransmitters? A. Dopamine B. Acetylcholine C. Glutamate D. GABA E. Serotonin
D Neurosciences
A patient presents with ptosis of the right eye, a dilated right pupil, double vision and left-sided hemiparesis without cognitive impairment. Which of the following brain regions is the likely lesion site? A. Basal ganglia B. Cerebral cortex C. Medulla D. Midbrain E. Pons
D Neurosciences
Among neurodevelopmental risk factors for aggression in male children, the evidence is strongest for a gene/environment interaction between maltreatment and genes for which of the following? A. Oxytocin B. Serotonin-transporter C. Dopamine-transporter D. Monoamine oxidase A E. Brain-derived neurotrophic factor
D Neurosciences
In which region of the brain does the morphology of neurons change the most during old age? A. Amygdala B. Hippocampus C. Perirhinal cortex D. Prefrontal cortex E. Lateral temporal cortex
D Neurosciences
Low cerebrospinal fluid levels of which of the following neurotransmitter metabolites has been most consistently associated with suicidality? A. Homovanillic acid (HVA) B. 3-0-methy1-Dopa (3-OMD) C. Gamma-hydroxybutyrate (GHB) D. 5-hydroxyindole acetic acid (5-HIAA) E. 3-methoxy-4-hydroxyphenylglycoI (MHPG)
D Neurosciences
The addition of methyl groups to nucleotides in the promoter region of a gene has what effect on transcription? A. Upregulates microRNAs B. Forms double-stranded RNA C. Opens chromatin to allow greater access to DNA D. Prevents transcription factors from binding to DNA E. Phosphorylates histone proteins associated with DNA
D Neurosciences
Which of the following best explains phenotypic differences between identical twins caused by environmental effects on genetic expression? A. RNA editing B. Somatic mutation C. RNA interference D. Epigenetic changes E. Post-translational modifications
D Neurosciences
Which of the following is a common form of sequence variation in the genome? A. Large deletions B. Microsatellite markers C. Simple tandem repeats D. Single nucleotide polymorphisms E. Simple sequence length polymorphisms
D Neurosciences
Which of the following neurohormones has a significant role in social bonding? A. Prolactin B. Corticotropin C. Somatostatin D. Oxytocin E. Melatonin
D Neurosciences
Which of the following neurotransmitters is most likely responsible for the rewarding nature of drugs of abuse? A. GABA B. Serotonin C. Glutamate D. Dopamine E. Norepinephrine
D Neurosciences
Which of the following statements about the brain's default mode network is accurate? A. Is activated by external stimuli B. Localizes primarily to the non-dominant hemisphere C. Uses much less energy than goal-oriented brain activity D. Is involved in reprocessing previously experienced stimuli
D Neurosciences
A psychiatrist is conducting a study to assess how a diagnosis of a chronic pain syndrome affects an individual's work performance as measured by the rate of errors. Which of the following statistical measures would be used to describe the probability that an individual with chronic pain will make an error compared to an individual who is pain free? A. Attributable fraction B. Effect size C. Odds ratio D. Relative risk E. Hazard ratio
D relative risk (RR), also sometimes known as the risk ratio, compares the risk of exposed and unexposed subjects, while the odds ratio (OR) compares odds. A relative risk or odds ratio greater than one indicates an exposure to be harmful, while a value less than one indicates a protective effect. Clinical psychiatry Research & scholarship literacy
A 65-year-old patient is brought to the emergency department with memory problems and confusion over the past two days. The patient is unable to follow simple commands and disoriented on examination with no apparent focal motor deficits. A magnetic resonance imaging (MRI) scan of the brain demonstrates increased signal in the bilateral medial temporal and orbitofrontal regions and electroencephalogram (EEG) shows periodic discharges over the left temporal lobe. Based on the information provided which of the following would be the likely treatment for this condition? A. Ceftriaxone B. Mannitol C. Heparin D. Acyclovir E. Prednisone
D Acyclovir - HSV Encephalitis Treatment is IV acyclovir, which must be given with adequate hydration to prevent crystal formation leading to Nephrotoxicity. Clinical neurology
Which of the following is the most commonly used cognitive therapy technique to uncover and modify automatic negative thoughts? A. Exposure B. Free association C. Guided imagery D. Socratic questioning E. Motivational interviewing
D. Socratic Questioning Clinical psychiatry Treatment
A diagnosis of Huntington disease would be best confirmed by which of the following methods? A. Karyotyping B. Genetic linkage analysis C. Fluorescent in situ hybridization D. Trinucleotide expansion analysis E. Mitochondrial DNA insertion analysis
D. Trinucleotide expansion analysis Karyotyping: Downs (trisomy 21), Edwards (18), Patau (13), Klinefelter (XYY), Turner (XO) Triad: Depression/Mood,Movement,cognitivedecline Neurosciences
Salivation in response to the smell of dinner represents which element of classical conditioning? A. Habituation B. Conditioned stimulus C. Conditioned response D. Unconditioned response E. Unconditioned stimulus
D. Unconditioned response Clinical psychiatry Behavioral & social sciences
The mesolimbic dopamine pathway includes which of the following structures? A. Hypothalamus B. Prefrontal cortex C. Substantia nigra D. Ventral striatum E. Periaqueductal gray matter
D. Ventral striatum ventral tegmental area in the midbrain to the ventral striatum of the basal ganglia in the forebrain Neurosciences
- Wellbutrin may cause what adverse effects?
Dizziness, agitation, dry mouth, insomnia, headache/migraine, nausea/vomiting, constipation, tremor, excessive sweating, blurred vision, tachycardia, confusion, rash, hostility, and auditory disturbance. Not known for prolonged QTc
- GAD:
Duloxetine, Escitalopram, Paxil, Effexor, Xanax
The Dx of dysthymia number of symptoms and duration?
Dysthymia is characterized by low mood for most of the days on most days for at least "Dy"-two years. It is diagnosed when there are at least "Dy" two symptoms present from the following list for at least two years: poor appetite or overeating, insomnia or hypersomnia, fatigue and low energy, low self-esteem, feelings of hopelessness and poor concentration.
A 65-year-old patient recently began to experience visual hallucinations of children playing. The hallucinations are fully formed, colorful and vivid, but without sound. The patient is not scared or disturbed by their presence, but rather amused. On examination, the patient has normal language and memory, normal cranial nerves, no weakness or involuntary movements and no sensory deficits. Deep tendon reflexes are symmetric. The results of cerebrospinal fluid (CSF) and routine laboratory tests, including toxic drugs screen, are normal. Which of the following is the most likely diagnosis? A. Advanced Alzheimer dementia B. Late onset schizophrenia C. Advanced dementia with Lewy bodies D. Progressive supranuclear palsy E. Posterior cerebral artery ischemia
E PCA strokes will primarily cause a visual field loss or homonymous hemianopia to the opposite side Clinical Neurology
Hearing impairment is a feature of which of the following chromosomal disorders? A. Rett syndrome B. Phenylketonuria C. Wilson disease D. Huntington disease E. Neurofibromatosis-2
E A. Rett syndrome- X link (girls) hand wringing, Dev Regress B. Phenylketonuria- C. Wilson disease- Cu in pee/poo, eyes, bones, liver D. Huntington disease E. Neurofibromatosis-2- 2 ears Shwanoma Neurosciences
A 75-year-old patient with a history of extensive tobacco smoking presents with subacute, progressive ataxic disorder over several months, Magnetic resonance Imaging (MRI) scan of the brain is unremarkable. Cerebrospinal fluid shows mild pleocytosis, As part of an extensive evaluation, the patient is found to have a positive anti-Yo antibody. Which of the following tests should be ordered next? A. Colonoscopy B. Genetic testing C. Lumbar puncture D. Electroencephalogram (EEG) E. Computed tomography (CT) scan of the chest
E Clinical neurology
Which of the following findings would support a diagnosis of ulnar neuropathy at the elbow? A. Decreased sensation in the thumb and second finger and weakness of thumb abduction B. Hypersensitivity to touch in the upper arm and weakness of raising the arm over the head C. Tenderness in the neck radiating to the back of the arm with decreased triceps reflex D. Impaired sensation on dorsum of hand and weakness of finger and wrist extension E. Loss of sensation in the fourth and fifth digit and weakness of intrinsic hand muscles
E Clinical neurology
A child displays appropriate behavior in order to avoid time outs. Which of the following features of operant conditioning is represented? A Extinction B Punishment C Sensitization D Positive reinforcement E Negative reinforcement
E Clinical psychiatry Behavioral & social sciences
A psychiatrist is consulted regarding a 43-year-old patient who complains of "a lump in my throat," headaches, bloating, back pain, diarrhea, chest pain, painful urination, and sexual indifference. The patient's complaints do not match the objective findings obtained from a thorough diagnostic workup. Which of the following would be best to recommend to the patient's primary care provider? A. Refer to a gastroenterologist B. Refer for intensive psychotherapy C. Prescribe an antipsychotic medication D. Tell the patient there is no medical illness E. Schedule regular visits with a physical examination
E Clinical psychiatry Consultation/collaborative innovative care Somatic symptom disorder DSM V criteria ≥ 1 somatic symptom(s) which are distressing to the patient or leads to a significant amount of disruption in the patient's life the patient experiences excessive thoughts, feelings, and behaviors in relation to their somatic symptoms or their health concerns these manifest as ≥ 1 of the following thoughts about the seriousness of their symptoms are disproportionate and persistent anxiety levels about their health or symptoms are persistently elevated concerns for their symptoms or health take excessive time and energy the somatic symptom must be persistent for ≥ 6 months although these symptoms don't have to always be present Treatment -have a single physician as the designated primary caretaker -schedule monthly visits and psychotherapy -avoid unecessary diagnostic testing unless indicated Conversion disorder (functional neurologic symptom disorder) DSM V criteria ≥ 1 symptom(s) of: altered sensory function, altered voluntary motor function, clinical findings are not consistent with recognized neurological or medical conditions the patient's symptoms are not better explained by another medical condition or medical disorder. the patient's symptoms causes significant distress or impairment in functioning or a need for medical evaluation Differential difficult to distinguish from functional neurologic disorder functional neurologic disorder is not often abruptly preceded by a life stressor Treatment -patient education and developing a therapeutic alliance (first-line) -cognitive behavioral therapy (CBT) in patients who do not respond to first-line treatment
Collaborative care interventions for psychiatric disorders in primary care settings include which of the following as one of its core components? A. Location of the psychiatrist in the clinic for consultations B. Collaboration with an outside psychiatric clinic for psychotherapy C. Involvement of the psychiatrist as a member of the primary care treatment team D. Presence of a psychiatric clinic in the immediate vicinity of the primary care clinic E. Care coordination and care management for patients by a non-physician provider
E Clinical psychiatry Consultation/collaborative integrated care
A 10-month-old infant is playing with a ball. When the ball is hidden under a blanket, the child moves the blanket and finds the ball. Which of the following aspects of cognitive development is reflected? A. Seriation B. Assimilation C. Conservation D. Accommodation E. Object permanence
E Clinical psychiatry Development & maturation
According to the predominant theories of adult development, a psychological task of mid-life is to: A. conduct a life review and reflect on past events. B. develop and maintain the capacity for intimacy. C. become a parent and transition to a position of authority. D. establish an adult work identity that becomes part of core identity. E. recognize the limitations of personal time and accept the inevitability of death.
E Clinical psychiatry Development & maturation
Infants appear to be more likely to develop an insecure attachment when they experience poor quality child care if they also have primary caretakers who: A. are working full time. B. are adoptive parents. C. are inexperienced in child care. D. differ in temperament from their child. E. are unresponsive to their infant's needs.
E Clinical psychiatry Development & maturation
Pretend play in preschool-age children depends upon children's increasing ability in which of the following skills? A. Emotion regulation B. Language acquisition C. Cooperation with peers D. Ability to follow parental rules E. Use of one object to represent another
E Clinical psychiatry Development & maturation
Which of the following is a typical phase-of-life problem in middle adulthood? A. Determining one's identity B. Achieving intimacy in relationships C. Choosing a sustainable career path D. Overcoming a sense of invulnerability E. Caring for both one's children and parents
E Clinical psychiatry Development & maturation
A 44-year-old patient presents with extreme fatigue and depression. Physical examination reveals darkening of skin and mucosa. Which of the following tests would most likely lead to the diagnosis? A. Porphobilinogen B. Vanillylmandelic acid (VMA) C. Thyroid-stimulating hormone (TSH) D 5-hydroxyindoleacetic acid (5-HIAA) E. Adrenocorticotropic hormone (ACTH) stimulation test
E Clinical psychiatry Diagnostic procedures
A patient presents for evaluation of "sleep problems." Though the patient is not aware of any physical discomfort, the bed partner notices that the patient experiences frequent "jerking" movements in both legs. The patient is referred for a sleep study, which reveals brief bursts of activity on electromyography lasting less than five seconds in duration and occurring every 20-40 seconds. Which of the following is the most likely diagnosis? A. Narcolepsy B. Sleep terror disorder C. Restless leg syndrome D. REM sleep behavior disorder E. Periodic limb movement syndrome
E Clinical psychiatry Diagnostic procedures
Which of the following is a methodologic limitation of epidemiologic cohort studies? A. Recall bias B. Ascertainment bias C. Reverse causality D. Control selection E. Loss to follow-up
E Clinical psychiatry Epidemiology
A patient with major depressive disorder is infrequently seen by the psychiatrist because the patient's depressive symptoms have responded very well to antidepressant therapy. The patient continues to see a social worker in an unaffiliated clinic for weekly therapy. The patient is fired from work and reports to the therapist feeling very depressed. This information is not conveyed to the psychiatrist, whom the patient has not seen in two months. The patient then dies by suicide, and the family sues both providers. Which of the following represents the legal responsibility in this situation? A. Only the psychiatrist may be responsible because the psychiatrist is a medical doctor. B. Only the psychiatrist may be responsible because the psychiatrist made the original diagnosis. C. Only the therapist may be responsible because the therapist saw the patient more recently, D. Only the therapist may be responsible because the therapist was obligated to tell the psychiatrist. E. Both providers may be responsible because both have a duty to know the condition of the patient.
E Clinical psychiatry Issues in practice
Immanuel Kant contributed which of the following to the development of psychiatry? A. Existential treatment concepts B. Theory of conflict in the psyche C. Empirical foundations for diagnosis D. Dialectical thinking and problem solving E. Philosophical foundations for clinical ethics
E Clinical psychiatry Issues in practice
Which of the following is an accurate statement about the Mental Health Parity and Addiction Equity Act of 2008 (MHPAEA)? A. Under MHPAEA, all health care plans musts include mental health and substance abuse benefits. B. Before MHPAEA, there were no federal regulations at all concerning mental health parity, C. MHPAEA applies to insurance plans provided by all employers, regardless of size or type of business. D. MHPAEA requires that all businesses that offer employee health insurance provide plans that cover mental health care. E. MHPAEA stipulates that financial requirements for mental health care be no more restrictive than those for medical/surgical care.
E Clinical psychiatry Issues in practice
Fear of abandonment is a core feature of which of the following disorders? A. Agoraphobia B. Delusional disorder C. Separation anxiety disorder D. Avoidant personality disorder E. Borderline personality disorder
E Clinical psychiatry Psychopathology & associated conditions
Paramedics were called to a party after security had to subdue an individual who was swinging at other party-goers and shouting, "You aren't gonna take me without a fight!" On examination the patient has nystagmus, hypertension, and decreased responsiveness to pain. Which of the following substances is the most likely source of the patient's behavior change? A. LSD B. Cocaine C. Marijuana D. Amphetamines E. Phencyclidine (PCP)
E Clinical psychiatry Psychopathology & associated conditions
Which of the following is increased following sexual reassignment surgery in patients transitioning from male to female? A. Suicide attempts B. Gender dysphoria C. Sexual dysfunction D. Relational problems E. General health problems
E Clinical psychiatry Psychopathology & associated conditions
A 12-year-old patient is referred for assessment of inattention and behavioral problems in the classroom and the home. Attention-deficit/hyperactivity disorder is diagnosed. The evaluation reveals a significant level of anxiety symptoms. Which of the following recommendations for initial treatment has the strongest evidence base? A. Alpha agonist monotherapy B. Serotonin specific antidepressant monotherapy C. Combined psychostimulant and alpha agonist D. Combined alpha agonist and behavior therapy E. Combined psychostimulant and behavior therapy
E Clinical psychiatry Treatment
Chronotherapy is an appropriate treatment for which of the following disorders? A Narcolepsy B Primary insomnia C Primary hypersomnia D Central sleep apnea E Circadian rhythm sleep disorder
E Clinical psychiatry Treatment
Transcranial magnetic stimulation for the treatment of depression targets which of the following brain regions? A. Thalamus B. Motor cortex C. Basal ganglia D. Parietal cortex E. Prefrontal cortex
E Clinical psychiatry Treatment
Which of the following antipsychotic medications is associated with decreased psychotic symptoms, decreased substance use, and increased abstinence in patients with schizophrenia and addictive disorder? A. Thiothixene B. Haloperidol C. Perphenazine D. Chlorpromazine E. Clozapine
E Clinical psychiatry Treatment
Which of the following mechanisms explains the primary effect of grapefiuit juice on the bioavailability of some drugs? A. Protein binding B. Fecal excretion C. Kidney excretion D. Stomach emptying E. Cytochrome metabolism
E Clinical psychiatry Treatment
Which of the following medications has the greatest potential to contribute to the development of galactorrhea in a patient on risperidone? A. Fluvoxamine B. Gabapentin C. Lorazepam D. Nefazodone E. Fluoxetine
E Clinical psychiatry Treatment
Which of the following statistical methods attempts to address the effects of participants dropping out of a study prior to completion? A. Odds ratio B. Meta-analysis C. Power analysis D. Positive predictive value E. Last observation carried forward
E Clinical psychiatry Research & scholarly literacy
A 72-year-old patient is admitted with renal failure. Over the course of several days the patient develops flaccid paralysis, difficulty speaking, dysphagia and impaired eye movements. Correction of which of the following electrolyte deficiencies can be associated with this disorder? A. Potassium B. Magnesium C. Chloride D. Calcium E. Sodium
E Guillan Barre and HypoNa go together Clinical neurology
An unmarried, nulliparous woman participates actively in the rearing of her siblings' children. According to Darwinian theory, such altruistic behavior would be explained by which of the following concepts? A. Sublimation B. Socialization C. Reciprocity D. Maladaptation E. Kin selection
E Kin Selection: evolutionary strategy that favors the reproductive success of an organism's relatives, even at a cost to the organism's own survival and reproduction. Sublimation- mature- socially unacceptable (aggression) into acceptable action. Using it in sports. Socialization- where an individual learns to adjust to a group Reciprocity- making mutually beneficial exchange with others Maladaptation- a trait that is more harmful than helpful Clinical psychiatry Behavioral & social sciences
A 5-year-old child presents with hyperuricemia, mental retardation, choreoathetosis and self-destructive biting of the lips and fingers. Which chromosome has been associated with this disorder? A. 4 B. 9 C. 15 D. 22 E. x
E Lech Nyhan CH X "put him on a lesch to stop self mutilation" HPRT1 gene result in the absence or deficiency of the enzyme hypoxanthine-guanine phosphoribosyl transferase on X Chromosome Neurosciences
134. Dysfunction of which of the following brain regions is most likely responsible for the perseveration evident in the clock drawing? A. Amygdala B. Hippocampus C. Caudate nucleus D. Precentral gyrus E. Prefrontal cortex
E Neurosciences
88. The role of glycine at the NMDA receptor can best be described as a(n): A. antagonist. B. partial agonist. C. inverse agonist. D. stearic modulator. E. obligate co-agonist.
E Neurosciences
A three-year-old girl presents for routine physical examination. Which of the following is the child expected to be able to do? A. Tie shoelaces B. Copy a square C. Copy a triangle D. Have imaginary friends E. Have a sense of herself as female
E Neurosciences
Decreased levels of which of the following neurotransmitters are most associated with depressed mood, poor sleep, and poor impulse control? A. Acetylcholine B. Dopamine C. Histamine D. Norepinephrine E. Serotonin
E Neurosciences
The presence of which of the following antibodies has been most consistently shown to substantially increase the risk of neuropsychiatric symptoms in lupus? A. Anti-Ro B. Anti-Hu C. Anti-Jo I D. Anti-thyroid E. Anti-phospholipid
E Neurosciences
Which of the following is a sign of an upper motor neuron injury? A. Diplopia B. Muscle atrophy C. Absence of Babinski signs D. Paresis with muscle flaccidity E. Hyperactive deep tendon reflexes
E Neurosciences
Which of the following is an example of a retrograde neurotransmitter? A. GABA B. Glutamate C. Dopamine D. Norepinephrine E. Endocannabinoids
E "End of Cannabis is so Retro" GABA- inhibitory, lowers HR, BP, relax to sleep Glutamate- Excitatory, Neuroplasticity, learn, memory Dopamine- reward, awake, coordination, motor control Norepinephrine (Epinephrine) from Tyrosine Serotonin- from Tryptophan ( Endocannabinoids- RetroGrade Neurosciences
A patient with autism is brought to the emergency department due to severe agitation, On examination, the patient has a fever of 38.20 C, heart rate 105, and blood pressure 160/95 mmHg. Leukocyte count is 18.1 103/gL, and creatine kinase is 15000 U/L The patient is noted to be mute, rigid, drooling, and hasn't eaten in 2 days. Chest x-ray and urinalysis are unremarkable. Current medications include fluoxetine, buspirone, methylphenidate, guanfacine, olanzapine, haloperidol, and valproic acid. Which of the following is the most likely diagnosis? A. Malignant catatonia B. Serotonin syndrome C. Anticholinergic toxicity D. Delirium secondary to infection E. Neuroleptic malignant syndrome
E NMS Difference in NMS (rigidity hyporeflexia)and SS is (clonus and hyperreflexia) Clinical psychiatry Treatment
Tracking patient satisfaction for accrediting agencies is characterized as which of the following types of administrative activity? A. Malpractice litigation B. Critical incident review C. Risk assessment review D. Ongoing utilization review E. Continuous quality improvement
E continuous quality improvement Clinical psychiatry Administration & systems
Which of the following is a first-line treatment for restless legs syndrome? A. Carbamazepine B. Clonazepam C. Fluoxetine D. Oxycodone E. Pramipexole
E. Pramipexole, Ropinarole Clinical neurology
Which of the following characteristics of major depression is indicative of greater overall severity? A. Fatigue B. Insomnia C. Weight gain D. Somatic complaints E. Psychomotor disturbance
E. Psychomotor disturbance Clinical psychiatry Psychopathology & associated conditions
- All things considered when should clinical assessment for abnormal involuntary movements be done FGA? SGA?
FGA- 6 mo general pop, 3 mo high risk SGA- 12 mo general pop, 6 mo high risk the APA guidelines (2004) remain a reasonable standard: "Clinical assessment of abnormal involuntary movements every 6 months in patients taking first-generation antipsychotics and every 12 months in those taking second-generation antipsychotics. In patients at increased risk, assessment should be done every 3 months and every 6 months with treatment using first- and second-generation antipsychotics, respectively (Patients at increased risk for developing abnormal involuntary movements include elderly patients and patients who experience acute dystonic reactions, other clinically significant extrapyramidal side effects, or akathisias)."
- Toluene is an aromatic hydrocarbon Female sniffing paint/glue, can make embryologic changes similar to what other syndrome?
Fetal Alcohol Syndrome. Toluene is used commonly in the manufacture of paints and organic compounds. It is abused by sniffing. Toluene embryopathy is developed in babies born to women who abuse toluene or exposed as a result of incidental occupational exposure during the first trimester of pregnancy. Features of toluene embryopathy include and overlap symptoms seem in fetal alcohol syndrome - growth retardation, microcephaly, micrognathia, small palpebral fissures, deep set eyes, low set ears, flat nasal bridge, and small fingernails. In older children, developmental delay, hyperactivity, language impairment, cerebellar dysfunction, and postnatal growth retardation become prominent.
- A patient with a past medical history of panic attacks and anxiety is brought into the ED by her husband as she has been stuporous and has been minimally responsive and extremely drowsy. The patient's husband mentions that his wife did have a panic attack today. or patient presents to the ED with impaired memory, poor concentration and extreme drowsiness. Pupils are not dilated on exam and the patient is minimally responsive. Of greatest concern is the patient's respiratory rate of 5/min. What should be given?
Flumazenil is a competitive benzodiazepine receptor antagonist that can be used as an antidote for benzodiazepine overdose but there are several contraindications to its use, including seizure disorder, as it can lower seizure threshold.
- PMDD:
Fluoxetine, Paxil, Sertraline Drospirenone/Ethinyl estradiol is the only combined oral contraceptive pill that is FDA-approved and effective for the treatment of premenstrual dysphoric disorder (PMDD) Premenstrual dysphoric disorder (PMDD), characterized by a disturbance in mood along with changes in appetite, sleep, concentration, interest, energy as well as physical symptoms like breast tenderness. These symptoms typically occur in the week prior to menstrual period. Fluoxetine or SSRI given 2 weeks prior to menstrual period is considered to be the gold-standard pharmacologic treatment.
- Prolactin-elevating drugs suppress gonadal hormone secretion and may enhance autoimmune proclivity, name medications that increase prolactin and treatment?
Fluphenazine, Haloperidol, Paliperidone, Risperidone, Metoclopramide, Domperidone, Clomipramine Two FDA-approved medications cabergoline and bromocriptine. Cabergoline may be better than bromocriptine at lowering prolactin levels. It also causes fewer side effects.
- In regard to laboratory tests
GGT and CDT are sensitive laboratory indicators of heavy drinking.
- What antiepileptics/mood stabilizers can Generate Tremors?
Gabapentin and Topiramate can cause tremors.
- Pt with dementia with severe renal and hepatic impairment, what medication should be avoided?
Galantamine is contraindicated in severe renal/hepatic impairment. Cholinesterase inhibitors
- Frataxin leads to Friedreich's ataxia
HEXA leads to Tay-Sachs disease, HEXB to Sandhoff disease, DMD to Duchenne's muscular dystrophy, and SMPD1 to Niemann Pick disease.
- Treatment for severe caffeine intoxication?
Hemofiltration and dialysis may be needed
- The clinical findings vertebral artery dissection are nystagmus
Horner syndrome, cerebellar abnormalities, and crossed sensory loss suggest a brain stem/cerebellar localization.
- Current guidelines for the treatment of enuresis in children?
Imipramine and desmopressin
-A young man is brought into the ED and has attempted to assault a nurse several times. He is extremely aggressive and becomes enraged when sudden movements or loud sounds are made. The patient is escorted to a dimly lit, quiet exam room where he becomes much calmer. On physical exam, the patient is agitated and has nystagmus. His blood pressure in the ED is 180/100 mmHg.
In general current clinical guidelines no longer recommend acidifying the urine in the management of PCP intoxication. But acidification is particularly dangerous when there is evidence of rhabdomyolysis, as it is likely to precipitate myoglobin in renal architecture and lead to acute renal failure. PCP- NMDA antagonist (Ketamine also) Tx: symptomatic only benzodiazepines antipsychotics (haloperidol) if benzodiazepines are not adequately sedating patient further management low stimulus environment restraints if needed to prevent patient from hurting self/others
- What can be seen in regards to the steady-state levels of tricyclic antidepressants (TCAs) in older adults vs younger populations?
Increased levels as both the renal excretion and hepatic blood flow are decreased in the elderly which results in overall reduced clearance in these patients. The results in a higher plasma level compared to younger patients, which may lead to increased potential therapeutic and adverse effects.
Erikson's Stages
Infancy: 0-18 mo- Trust vs Mistrust Toddlerhood: 18- 3yo: Autonomy vs Shame Preschool: 3-6: Initiative vs Guilt School age: 6-puberty: Industry vs Inferiority Adolescence: teens-20s: Identity vs Role Confusion Young Adulthood: 20s-40s: Intimacy vs Isolation Middle adulthood: 40s-60s: Generativity vs Stagnation Late Adulthood: 60s +: Ego/Integrity vs Despair
- Fluoxetine will raise the levels of TCA medications by inhibition of which CYPs?
Inhibition of CYP2 D6 and C19. Therefore levels of tricyclic antidepressants, which are metabolized by this CYP2D6 as well as others. By inhibiting CYP2D6, fluoxetine will increase TCA levels.
- Should Lithium be continued in breastfeeding mom?
It is contraindicated during breastfeeding due to risk of accumulation in the neonatal kidneyas it does pass through breast milk.
-Patient taking Fluvoxamine then is started on antibiotic (ciprofloxacin) what will happen to serum level of Luvox and why?
It would rise with the addition of an inhibitor of this enzyme such as ciprofloxacin. Fluvoxamine is a substrate of P450 1A2 as well as 2D6. CRAP GPs can be used to easily remember common CYP450 inducers. Carbemazepines Rifampicin Alcohol Phenytoin Griseofulvin Phenobarbitone Sulphonylureas SICKFACES.COM can be used to easily remember common CYP450 inhibitors. Sodium valproate Isoniazid Cimetidine Ketoconazole Fluconazole Alcohol & Grapefruit juice Chloramphenicol Erythromycin Sulfonamides Ciprofloxacin Omeprazole Metronidazole
- What can be used for hypertensive crisis.
Labetolol
- most common drug causing hypernatremia?
Lithium (Li)
- Mania:
Lithium, Depakote, Seroquel, Zyprexa, risperidone, Abilify, Saphris, Geodon, chlorpromazine, carbamazepine
- What is the first step in suspected Meningitis?
Lumbar puncture is indicated immediately in suspected meningitis.
- Patient taking certain antidepressant class may cause hypotension?
MAOi's may cause hypotension.
- Norepinephrine is metabolized into its primary metabolites VMA or a conjugated form of 3-Methoxy-4-hydroxyphenylglycol (MHPG
MOPEG), low levels of which are associated with anorexia nervosa and pathological gambling.
- Ethnic population at greatest risk of developing metabolic syndrome?
Mexican-Americans are at the greatest risk of developing metabolic syndrome.
- MoCA for cognitive screening
Mood Disorder Questionnaire for mania screen, Epworth Sleepiness Scale for OSA
- PregNant woman drinking, what medication should be given?
Naltrexone is the medication of choice to reduce cravings in pregnant women.
- When is Sodium oxybate prescribed?
Narcolepsy only when there is catalepsy. -Another name for Sodium Oxybate (Xyrum/ GHB) DSM V criteria In order to make a diagnosis of narcolepsy, the individual must have symptoms daytime sleepiness occurring at least three times a week over the past 3 months. In addition, one of the following must be present: 1.Hypocretin deficiency (orexin A) in CSF 2.Episodes of cataplexy occurring at least several times a month 3.REM sleep latency of fewer than 15 minutes or two or more sleep-onset REM periods (SOREMPs) and a mean sleep latency of fewer than 8 minutes.
-What is Natalizumab used for and it's risk factor?
Natalizumab is used for Crohn disease and multiple sclerosis. However There is an increased risk of JC virus infection and the development of progressive multifocal leukoencephalopathy (PML) (due to JC virus infection).
- Creutzfeldt-Jakob disease can result from corneal transplants. Different neurological investigations are done for diagnosing the disease. Brain biopsy or autopsy is the only definitive confirmatory investigation while others provide a suggestive edge towards diagnosis. Cerebrospinal fluid studies often show 14-3-3 protein in prion's disease. Electroencephalogram (EEG) shows typical periodic sharp, synchronous triphasic waves at a rate of 0.5-2 Hz at a certain point during the course of the disease but is not present in all patients.
Neuroimaging comprises of magnetic resonance imaging (MRI), specifically diffusion-weighted imaging (DWI) and fluid-attenuated inversion recovery (FLAIR) showing multifocal gray matter hyperintensities in subcortical and cortical regions of the brain. This reflects the pattern of symptoms of the disease comprising of movement disorders like myoclonus, chorioathetosis, and ballismus. Blood testing of serum for prion proteins is still under ongoing research. The challenges in the detection of prion proteins in blood serum stem from its complex and diverse biochemical structure. On the other hand, detection in brain tissue, mostly in advanced disease has yielded definite diagnostic success.
- TMS - approved for TRD
OCD, pain with migraines; useful for AH
- The best-studied antipsychotic for pregnant women?
Olanzapine
EEG changes - olanzapine or risperidone and are frequently dose dependent -barbiturates or benzodiazepines -Opioids.
Olanzapine/Risperidone- Nonspecific changes Benzos/Barbiturates- increased Beta activity Opioids- decreased Alpha activity triphasic waves- encephalopathy, metabolic, toxicity
Types of play during Preoperational Stage
PAC play- Pretend Play Parallel play - 2y - play w/similar toys but separately Associative play - 3y - share toys but play separately Cooperative play - 4y - share toys and interact w/each other
- All patients presenting with suspicion of hydrocarbon exposure what is the first step in treatment?
Patients should immediately be placed on a cardiac monitor (because dysrhythmias are the most common cause of death) as well as pulse oximetry. A chest X-ray is indicated in any patient suspected of inhalation or aspiration of hydrocarbons. However initial radiograph may be normal although patient is showing symptoms. On the other hand, initially asymptomatic patients may have abnormal radiographs later during their course. About six hours after exposure, patients will experience symptoms or have changes in chest radiography or pulse oximetry. So, keeping the patient in observation for six hours and discharging if the patient remains asymptomatic with normal chest x-ray is the appropriate approach to manage an asymptomatic patient with hydrocarbon exposure.
- What population is particularly vulnerable to developing tardive dyskinesia?
Patients with developmental disabilities are
- PTSD:
Paxil, Sertraline, Zoloft
- Various studies have found a small increase in risk of what in infants whose mothers use SSRI in 2nd and 3rd trimester?
Persistent Pulmonary Hypertension of Neonate (PPHN) of the newborn as a result of SSRI exposure in the second and early third trimester. PPHN is a serious breathing condition in a newborn in which lung vessels are not open wide enough meaning that oxygen and blood flow is restricted. One symptom is that the baby's skin is blue.
- A 21-year-old male is brought into the ED by the police for an altercation. Last night, the patient was at a party and seemed much more active than usual according to his girlfriend. He punched another male at the party in the face claiming that he was hitting on his girlfriend. On exam, you see an agitated young male with dilated pupils, and his pulse is 128/min.
Phenoxybenzamine (a-1 blocker) can be used for cocaine toxicity which is evident by his severe anxiety, agitation, dilated pupils and physical exam findings (high blood pressure and elevated heart rate). The patient has severely elevated blood pressure, which must be controlled. Cocaine toxicity results in unopposed alpha- and beta-adrenergic stimulation. DO NOT GIVE B-blocker.... If a beta-blocker is given first, there will be excessive alpha-adrenergic stimulation, which will cause further elevation of the patient's blood pressure. treatment: pharmacologic antipsychotics (haloperidol) benzodiazepines antihypertensives (labetalol - need alpha-1 blockade) vitamin C - promotes excretion non-pharmacologic do not restrain patients-- may result in rhabdomyolysis
- Methyl alcohol poisoning - formate toxicity leads to infarction or hemorrhage where?
Putamen Tx: give Fomepizole (Foamy Piz is all) to block alcohol dehydrogenase from making Formaldehyde/Glycolaldehyde -a part of the lenticular nucleus in the basal ganglia of the brain. It receives input from the motor cortex and is involved in control of movements - involved in learning and motor control, including speech articulation, language functions, reward, cognitive functioning, and addiction
- All FGAs are associated with?
QT prolongation
- LBD: clinical diagnosis with cognitive decline with loss of function and 2/4: VH
REM sleep behavior disorder, fluctuating cognition, and parkinsonism; PET scan and reduced DA uptake in basal ganglia on SPECT (DaTSCAN) most specific tests
- Hydrocortisone has been shown to do what after trauma?
Reduce the severity of symptoms and decrease the risk of developing PTSD
Miller Relationship
Relational theory Miller Relationship between pt and therapist achieves intimacy
- 61-year-old male presents with left-sided hand weakness and trouble with walking. He is not sure why these symptoms occur. On physical exam, tongue fasciculations are appreciated. He has slow speech. The left upper extremity shows forearm atrophy and depressed reflexes. The right lower extremity is hypertonic, with 3+ reflexes, and positive Babinski sign, diagnosis, treatment and MOA of treatment?
Riluzole is a neuroprotective drug that blocks glutamatergic neurotransmission in the CNS used for ALS. ALS: Upper & Lower MN dz EMG: widespread denervation and fibrillation potential Upper motor neuron (UMN) signs spasticity or stiffness hyperreflexia dysarthria dysphagia lower motor neuron (LMN) signs atrophy fasciculations hyporeflexia weakness
- Atypical antipsychotics used for augmented pharmacotherapy for the treatment of PTSD?
Risperidone, especially with co-occurring psychotic symptoms.
- Patient taking carbamazepine and starts to have slurred speech, double vision, ataxia, and somnolence., Dx?
SIADH is a recognized side effect of carbamazepine (CBZ) therapy even with therapeutic doses. Elevated CBZ levels are more likely to manifest as slurred speech, double vision, ataxia, and somnolence. Syndrome of inappropriate antidiuretic hormone secretion (SIADH) is a condition in which the body makes too much antidiuretic hormone (ADH). This hormone helps the kidneys control the amount of water your body loses through the urine. SIADH causes the body to retain too much water thus causing hyponatremia.
- Rash in the setting of medication initiation with Lymphadenopathy is likely?
SJS, lymphadenopathy helps differentiate a benign rash from SJS. SJS <10% BSA involvement TEN >30% more severe
- Psychotropic-induced thrombocytopenia is a potential side-effect of what medications?
SSRI antidepressants. Selective serotonin re-uptake inhibitors (SSRI) have inhibitory effects on 5-HTT and 5-HT receptors of platelets, thereby diminishing platelet aggregation. There have been many reports of SSRI causing reductions in platelet count and sometimes resulting in thrombocytopenia. Low platelet levels can have causes that aren't due to underlying disease. Examples include pregnancy, altitude, or medication side effects
- Treatment for skin-picking disorder
SSRIs, naltrexone, lamotrigine, and N acetylcysteine.
- The first line anti-depressants that have the lowest level in breast milk and are safer to use include?
Sertraline, paroxetine, imipramine, and nortriptyline. Desipramine and doxepin belong to the group secondary and tertiary TCAs respectively, and they are considered less safe compared to sertraline. Fluoxetine has been shown to accumulate in nursing infants due to its long half-life. Medications to avoid are monoamine oxidase inhibitors (MAOIs), like tranylcypromine.
How do you diagnose an "other specified depressive disorder"?
Short-duration depressive episode, is an This is characterized by 4-13 days of depressed mood with the presence of any one of the 8 other symptoms required for the diagnosis of a major depressive episode.
What are common neuroanatomy findings of PTSD?
Small hippocampus volume, as well as abnormal volumes of the amygdala, insular cortex, medial prefrontal cortex, and dorsal prefrontal cortex
-What does smoking do to Asenapine, Clozapine and Olanzapine?
Smoking induces CYP1A2 thus reducing Clozapine, Asenapine, and olanzapine levels as are substrates of isoenzyme CYP1A2. Polycyclic aromatic hydrocarbons present in the smoke induces CYP1A2 through aromatic hydrocarbon receptors. Olanzapine is also metabolized by CYP3A4 which is not affected by smoking.
- What does St. John's Wort do to the efficacy of oral contraceptives.
St. John's Wort decreases the efficacy of oral contraceptives. Others: Ketoconazole, Rifampin, Griseofulvin, Anitepileptics (carbamazapine,
- Capitation
System of payment used by managed care plans in which physicians and hospitals are paid a fixed, per capita amount for each patient enrolled over a stated period regardless of the type and number of services provided; reimbursement to the hospital on a per-member/per-month basis to cover costs for the members of the plan. is described as a set amount of money that is paid to the physician in advance for each patient per unit time in return for their health care services. Fee-for-service schedule is incorrect as it a reimbursement method which is described as payment of a preset fee for each specific service or procedure performed.
- 43-year-old woman is brought to the emergency room by her sister after ingesting an unknown medication. Her sister states that she rushed to the patient's house after the patient called her saying she took "a handful of pills", but found the patient unresponsive when she arrived. She knows the patient is being treated for fibromyalgia and major depressive disorder but is unable to recall her sister's medication regimen. The patient's temperature is 100.9°F (38.3°C), blood pressure is 92/65 mmHg, pulse is 120/min, and respirations are 15/min. She is deeply somnolent but opens her eyes briefly to sternal rub. Her pupils are dilated, equal, and reactive. Her skin is warm and flushed. Bowel sounds are absent. Her ECG is shown in Figure A (shows arrythimia) What is the mechanism of action of the most likely ingested substance?
TCA. (serotonin and norepinephrine reuptake inhibition) Tricyclic antidepressants work by blocking the reuptake of serotonin and norepinephrine; overdose presents with signs of anticholinergic toxicity, coma, convulsions, and QRS prolongation. Overdose of SSRIs can cause serotonin syndrome, characterized by neuromuscular hyperactivity (tremor, hyperreflexia, clonus), hyperthermia, and agitation. QRS changes would not be expected. Physostigmine is a cholinergic agonist that could be used to treat anticholinergic toxicity.
- Anticholinergic delirium may occur following overdose with which anticholinergic agent?
TCAs; patients present with dry mucous membranes, hot, dry flushed skin, anhidrosis, blurred vision from mydriasis, fever, absent bowel sounds, and urinary retention. Altered mental status manifests as confusion and delirium with pressured, incoherent speech, agitation and visual and/or auditory hallucinations.
- In patients with neurocognitive disorder due to frontotemporal lobar degeneration
TDP-43 proteinopathy is often seen on autopsy.
Specificity
TN / TN + FP 92% spec means 8% FP
Sensitivity
TP / TP + FN 95% sens means 5% FN
What Two medicaTions my cause hyponaTremia?
Tegretol (carbamazepine) and Trileptal (oxcarbazepine) may cause hyponatremia.
-How can a brain thallium-201 SPECT scan can be used as a sensitive and specific method to differentiate a CNS lymphoma from cerebral toxoplasmosis?
Thallium is more readily taken up by hypermetabolic cancer cells, and can indicate CNA lymphoma.
- What are the 4 primary symptoms of Parkinson's disease?
The 4 primary symptoms of Parkinson's disease are: Tremor (trembling in hands, arms, legs, jaw, and face) Rigidity (stiffness of the limbs and trunk) Bradykinesia (slowness of movement) Postural Instability (impaired balance and coordination)
- Pt on rivastigmine, memantine, galantamine, and starts having gastrointestinal disturbance; vomiting, diarrhea, nausea, what is the next step?
The current recommendation of management is the conversion of oral rivastigmine to administration via a transdermal patch or switch to another acetylcholinesterase inhibitor (such as donepezil.) Switching to a transdermal patch has proven to decrease the incidence of gastrointestinal disturbances associated with oral intake while maintaining the fair administration of daily dosage.
- Patient taking new medication for bipolar disorder, with excessive thirst. Excessive urine production (polyuria). Symptoms of electrolyte imbalances including, unexplained weakness, lethargy, muscle pains, and irritability. What is going on and treatment?
Treat with Amiloride (potassium sparing diuretic). Which reduces lithium uptake in the cells of the collecting duct improving responsiveness to ADH-stimulated translocation of AQP2 to the membrane of the principal cells. Lithium inhibits ADH-stimulated translocation of cytoplasmic urinary aquaporin 2 (AQP2) to the apical membrane on the collecting duct. Less translocation of AQP2 to the cell's membrane decreases the kidney capacity to reabsorb water. Resulting in the abnormal excretion of dilute urine.
- Atropine induced psychosis is accompanied by anticholinergic effects such as dry mouth, decrease gastrointestinal motility, etc. treat with?
Treat with P-Atropine
- What is VPA MOA?
VPA increases synaptic levels of GABA by blocking GABA transaminase.
- Who is more prone to tardive dyskinesia?
Young males are
- Panic disorder:
Zoloft, Fluoxetine, Sertraline, Paxil, Effexor, Xanax, Klonopin
AR
a / (a+b) - c / (c+d)
- Parallel play is a stage of kids play in which a child plays besides the other children without interacting with them. He usually watches and tries to copy their play but not ready to interact with them directly. This stage usually lasts from 2 - 3.5 years of age. In associative play
a child may play with other children but will not be working towards an achievable goal or making connections with other children. It lasts from 3 - 4 years of age. Cooperative play is a stage in which children learn to play with each other while using social skills and interacting with peers. A child's age during this stage is about 4-6 years. In onlooker play, a child stares and observes the other children's play but doesn't try to play by himself. It usually precedes the parallel play. Self-stimulatory play includes certain types of behaviors such as arm flapping, spinning toys, twirling, etc., and it is generally associated with autism.
- Medications currently approved by the FDA for the treatment of insomnia include benzodiazepine receptor agonists (BZRAs)
a melatonin receptor agonist (ramelteon), and a tricyclic antidepressant (doxepin). Zaleplon is a BZRA with rapid onset and short duration of action, making it ideal for sleep-onset insomnia. It also has a relatively better side effect profile and less residual sedation than other drug options due to its short half-life (approximately 1 hour). Eszopiclone and Zolpidem are also BZRAs, but have longer half-lives than zaleplon and can result in impaired cognitive performance the following day. Flurazepam is a long acting benzodiazepine, and temazepam is an intermediate acting agent, both of which can result in residual daytime sedation due to their long half-lives.
- Ethics-based requirements of third-party evaluations commonly reflect the understanding that although a traditional treatment relationship does not exist
a third-party evaluation does establish a limited doctor-patient relationship. Disability evaluations are not confidential. However, the consensus of the duties of the psychiatrist in these evaluations include, at a minimum, to maintain confidentiality to the extent possible, to disclose significant findings, and not to cause harm to the individual. Lawsuits based on principles of medical malpractice and ordinary negligence are less common than in clinical practice but are increasing in frequency.
RR
a/(a+b) / c/(c+d) Relative risk (RR) risk of disease in exposed group/risk of disease in unexposed group = [a/(a+b)] / [c/(c+d)] used in cohort studies or other studies where total population is known RR > 1 exposure is associated with increased risk of disease RR < 1 exposure is associated with decreased risk of disease RR = 1 no association between exposure and disease Attributable risk (AR) "How much greater risk is present in the exposed group than the unexposed?" risk of disease in exposed group - risk in unexposed group = a/(a+b) - c/(c+d) Attributable risk percent (ARP) "What percent greater risk is present in the exposed group than the unexposed?" (risk of disease in exposed group - risk in unexposed group)/risk in exposed group = [a/(a+b) - c/(c+d)] / [a/(a+b)] Absolute risk reduction (ARR) "How much risk is reduced by the intervention (or exposure)?" risk in control group - risk in intervention group = c/(c+d) - a/(a+b)
Odds ratio
a/c / b/d = ad/bc Odds ratio (OR) odds of having disease in exposed group/odds of having disease in unexposed group = ad/bc primarily used in case control studies can be used in cohort studies when outcome is rare OR approximates RR for rare outcomes OR > 1 odds of developing disease are greater in exposed group OR < 1 odds of developing disease are reduced in exposed group OR = 1 odds of developing disease are equal in exposed and unexposed group Relative risk (RR) risk of disease in exposed group/risk of disease in unexposed group = [a/(a+b)] / [c/(c+d)] used in cohort studies or other studies where total population is known RR > 1 exposure is associated with increased risk of disease RR < 1 exposure is associated with decreased risk of disease RR = 1 no association between exposure and disease Attributable risk (AR) "How much greater risk is present in the exposed group than the unexposed?" risk of disease in exposed group - risk in unexposed group = a/(a+b) - c/(c+d) Attributable risk percent (ARP) "What percent greater risk is present in the exposed group than the unexposed?" (risk of disease in exposed group - risk in unexposed group)/risk in exposed group = [a/(a+b) - c/(c+d)] / [a/(a+b)] Absolute risk reduction (ARR) "How much risk is reduced by the intervention (or exposure)?" risk in control group - risk in intervention group = c/(c+d) - a/(a+b)
- The best diagnostic test to measure Acute Intermittent Porphyria is the urine porphyrin levels. The patient given in the scenario is suffering from acute intermittent porphyria (AIP). The disorder is due to an abnormality in hemoglobin metabolism. Patient presents with psychiatric and neurologic signs. Patients of AIP also have complaints of constipation
abdominal discomfort, neuropathies but they do not have any cutaneous rash. AIP has an autosomal dominant pattern of inheritance and manifests between the age of 18 to 40. Patient's exhibit episodes which are triggered by certain medications like benzodiazepines, estrogens, and barbiturates. Diclofenac amplifies these episodes. Psychiatric symptoms include anxiety, depression, psychosis, and delirium. Urine porphobilinogen are elevated during attacks and can help in establishing a diagnosis.
- The trochlear
abducens and hypoglossal nerves arise from the basal plates of the brainstem. The origin cells of these nerve are located in the somatic efferent column.
- DeGeorge/velocardialfacial syndrome (CATCH-22): cardiac abnormality (ToF)
abnormal facies (microcephaly, micrognathia), thymic aplasia, cleft palate, hypocalcemia/hypoparathyroidism; speech and developmental delays, 30% have psychosis, 25% develop schizophrenia; AD inheritance
Type II error
accepting the null hypothesis, saying there is no difference, when a treatment effect truly exists.
- The following drugs can lower serum lithium concentrations by increasing urinary lithium excretion:
acetazolamide urea, xanthine preparations, and alkalinizing agents such as sodium bicarbonate.
- Analgesia induced by opioids works through neuronal hyperpolarization and suppression of neuronal excitability. This process is dependent on heterotrimeric G protein-dependent mechanisms which have been shown in rodent models and in vitro models showcasing the role of MOR-dependent analgesia. The regulation of neuronal firing is carried out by Gai-mediated inhibition of cAMP production
activation of G-protein coupled inwardly rectifying potassium channels (GIRK) and Gbg-dependent inhibition of L-type calcium channels.
- The consortium on Dementia with Lewy Bodies has developed clinical criteria to improve sensitivity and specificity of diagnosis. Two of three core clinical characteristics
addition to dementia, must be present. The three core clinical criteria are fluctuations in cognitive function with varying levels of alertness and attention; visual hallucinations; and Parkinsonism. Visual hallucinations occur in up to two thirds of persons with Lewy body disease and are relatively rare in patients with Alzheimer's disease. Other symptoms in Neurocognitive Disorder with Lewy Bodies, delusions, rapid eye movement sleep disorder, unexplained syncope and neuroleptic sensitivity.
- Attention-deficit/hyperactivity disorder (ADHD) and conduct disorders are risk factors for what?
aggression, and impulsivity have consistently been reported as clinical risk factors for later bipolar disorder in prospective studies.
- Females with bipolar disorder have a higher risk of developing what more so than males?
alcohol use disorder as compared to males.
- Social communication disorder is characterized by difficulty in using language and communicating socially. Individuals with this condition are unable to understand or follow verbal and nonverbal social cues or rules. This manifests as deficits in greeting others appropriately and the inability to change language according to the situation or person they are conversing with. They also face problems in sticking to the rules when it comes to telling stories or having a conversation (such as responding to the cues made by the listener and making adjustments to the conversation or taking turns to talk). They are unable to understand implicit meanings or non-literal use of words like jokes and idioms
all of which can be seen in this case.
- Viral meningitis
also known as aseptic meningitis, is inflammation of the leptomeninges as a manifestation of CNS infection. CSF in viral meningitis is often characterized by elevated protein levels, clear CSF, opening pressure that is normal or elevated, glucose levels that are normal or slightly low, and WBC counts elevated to between 10-10,000 with majority Viral Meningitis: LympVocytes. and Protein Normal CSF values: clear fluid, opening pressure of 10-20 cm H2O, WBC count 0-5 cells/microL, glucose >60% of serum glucose, and protein level < 45mg/dL.
- The most effective intervention for trichotillomania
although still with limited efficacy, is habit reversal training, which helps patients replace repetitive distressing behaviors with competing behaviors that are more socially acceptable, less disfiguring, and less distressing.
- Anti-NMDA receptor encephalitis: flu-like illness followed by psychosis
amnesia, AVH, SI, dyskinesias, respiratory dysfunction; T2/FLAIR intensity in mesial temporal lobes and limbic systems
- There are some notable neurophysiological differences between unipolar and bipolar depression which could explain why treatment is different. There is a reduction in the number of connections between pregenual anterior cingulate cortex
amygdala and dorsomedial thalamus in patients with bipolar depression as compared to unipolar depression.
- Amok is a condition in which an individual exhibits two phases
an outburst of violence and a dissociative phase.
- Tasimelteon is the only drug approved for the treatment of Non-24 by the US FDA
and it is a potent and specific Melatonin (MT1 and MT2) receptor agonist with a greater correspondence for the MT2 receptor. It is orally well-tolerated among totally blind patients with Non-24. Non-24 is a circadian rhythm disorder and occurs because our internal clock is not synched with light and dark cycles of the day and night. Most individuals with this disorder are totally blind. That's because your internal clock gets its cue from seeing light. But sometimes people who have normal vision also get it.
- Citalopram is a substrate for 2C19 and 3A4
and paroxetine for 2D6.
What is criteria for MDD with Melancholic features?
anhedonia + 3/6: despondency, despair, depression worse in the morning, early-morning awakenings, psychomotor changes, anorexia/weight loss, excessive/inappropriate guilt
- Trazodone has never been approved for insomnia, but is often used and works as an?
antagonist of H1- and α1-receptors 'Tr H a zod-1' (T anTagonist to H and a 1) and also inhibits serotonin reuptake. ...but may cause Torsades de Pointes.
- Damage to the hippocampi can result in what type of amnesia?
anterograde amnesia.
- Brain fag is an
anxiety state exhibited by students mainly in Africa when they find it difficult to tackle challenges appropriately.
- Late life migraine accompaniments often have both "positive" and "negative" symptoms. They most commonly consist of scintillations or other visual displays. Next in order are paresthesias
aphasia, dysarthria, and paralysis. There is a "build-up" of these accompaniments, especially the visual display (they start small and build up in size and intensity). There may be a "march" of the paresthesias (they start in one location and spread to other parts of the body) usually over minutes. There is often a progression from one accompaniment to another, often with a delay. There tend to be many spells which are very similar or occur in a stereotype fashion. Headaches may or may not occur. The episodes last on average from about 15 to 25 minutes. Characteristically, these spells occur in midlife or elderly and often occur in "flurries." In general, the clinical course is benign; however, similar symptoms may occur early with cerebral amyloid angiopathy.
- The 4 primary symptoms of Parkinson's disease are tremor (trembling in hands
arms, legs, jaw, and face), rigidity (stiffness of the limbs and trunk), bradykinesia (slowness of movement), and postural instability (impaired balance and coordination).
- One mechanism of action of Acamprosate is?
as a blocking agent for NMDA receptors.
- Frontotemporal dementia is characterized by a progressive change in an individual's personality and behavior
as well as variable degrees of cognitive and language impairments. Several mutations have been associated with frontotemporal lobar degeneration (FTLD), including mutations in the microtubule associated protein tau (MAPT), the granulin gene (GRN), and the C90RF72 gene. The mutation in MAPT is a common cause of frontotemporal dementia and Parkinsonism linked to chromosome 17 (FTDP-17), and follows an autosomal dominant pattern of inheritance.
- Taking Acetylsalicylic acid (ASA) with VPA can cause what? Taking cholestyramine + VPA can cause what? Taking Carbamazepine or phenytoin + VPA can cause?
aspirin, can reduce the elimination of valproic acid and result in elevated blood concentrations of valproic acid. Cholestyramine can reduce the absorption of valproic acid from the intestine. Carbamazepine and phenytoin can increase the elimination of valproic acid and thus reduce blood concentrations.
- Microglial cells are part of the mononuclear phagocytic population and are derived from mesenchymal cells. Neuroepithelial cells give rise to oligodendrocytes
astrocytes, and ependymal cells. Glioblasts are derived from neuroepithelial cells and give rise to astroblasts and oligodendroblasts. Astroblasts are derived from glioblasts and give rise to astrocytes. Neuroblasts give rise to neurons.
- Epileptic myoclonus includes progressive degeneration disorders of the nervous system in which myoclonus may occur in association with cognitive impairment
ataxia, and multiple seizure types. Examples include myoclonic absence, infantile spasms, and Lennox-Gastaut syndrome.
- Friedreich's ataxia - AR disease with mutation on chromosome 9 encoding frataxin; GAA trinucleotide repeat; weakness
ataxia, high arches, scoliosis, cardiac problems
- Lennox-Gastaut syndrome occurs in children and is defined by the following triad: multiple seizure types (usually including generalized tonic-clonic
atonic, and atypical absence seizures); EEG showing slow (<2.5 Hz) spike-and-wave discharges and a variety of other abnormalities (shown above); and impaired cognitive function in most but not all cases This syndrome is associated with CNS disease or dysfunction from a variety of causes, including developmental abnormalities, perinatal hypoxia/ischemia, trauma, infection, and other acquired lesions. The prognosis in these patients is often poor due to the underlying CNS disease and the physical and psychosocial consequences of severe, poorly controlled epilepsy.
- The DSM-5 revised the PTSD diagnostic criteria to include a history of exposure to a traumatic event that meets specific stipulations and symptoms from each of four symptom clusters: intrusion
avoidance, negative alterations in cognitions and mood alterations in arousal reactivity. The sixth criterion concerns duration of symptoms; the seventh assesses functioning; and the eighth criterion clarifies symptoms as not attributable to a substance or comorbid medical condition. Two specifications are noted including delayed expression and a dissociative subtype of PTSD, the latter of which is new to DSM-5. In both specifications, the full diagnostic criteria for PTSD must be met for application to be warranted. Duration of the symptoms must be more than 1 month. Symptoms can occur after the trauma, and the specifier "with delayed expression" is given if the onset of symptoms is at least 6 months after the trauma.
- Stiff person syndrome is an autoimmune disorder. It presents with exaggerated lumbar lordosis
back and lower limb spasms and stiffness. Patients can be rendered immobile; however, they tend to communicate their pain due to the spasms and their episodes are brought on due to emotional stimuli.
- The most commonly found structural changes in the brains of patients suffering from depression include abnormal hyperintensities in subcortical regions. These hyperintensities are particularly seen in the periventricular area
basal ganglia and thalamus.
Why does Fluoxetine Raise Risperidone levels?
because it is metabolized only by CYP2D6 and fluoxetine inhibits D6.
- Progressive supranuclear palsy (PSP) is a degenerative neurologic disorder characterized by a combination of cognitive
behavioral, balance, and visual symptoms. Among the most common signs are difficulty in the voluntary movement of the eyes in the vertical plane and impairment in voluntary eye saccades. Both difficulties with eye movement and balance are thought to contribute to sudden falls.
- Utilization behavior is a neurobehavioral disturbance in which the visual cue of an object stimulates the subject to pick up and begin to perform the appropriate behavior proper to the object
but inappropriate with respect to time and circumstance. This is a disturbance of impulse control that has been linked to the frontal lobes.
- Ethnicity is a factor in the occurrence of CYP2D6 variability and should be taken into account. The prevalence of CYP2D6 poor metabolizers is approximately 6% to 10% amongst white populations
but is lower in most other ethnic groups, such as Asians (2%). However, in blacks, the frequency of poor metabolizers is greater than for whites.
- Copper deficiency is most commonly found in patients who have undergone gastric bypass surgery
but it can also occur in people taking excessive amounts of zinc as zinc and copper are absorbed in the GI tract via the same transporter.
- Pyridoxine's benefit is supported by a well-designed trial. Dose is up to 400 mg every day. Vitamin E has been evaluated in numerous studies
but its benefit remains to be conclusively established. Dose between 400-1600 iu every day.
- Amoxapine is a tetracyclic compound FDA approved for treating depression that may cause what adverse effect?
but with a metabolite, 7-hydroxyamoxapine, that has a dopamine receptor blocking effect, giving it an action similar to an atypical antipsychotic medication. It may cause tardive dyskinesia.
- What does Bupropion do to TCA levels?
can elevate nortriptyline levels.
- Celexa may cause what adverse effects?
cause hyponatremia and increase the risk of GI bleeds
- Denervation or dysfunction of small nociceptive fibers leads to enhanced nerve excitability
causing neuropathic pain.
- Diabetic neuropathy is one of the most common causes of small fiber neuropathy. Denervation or dysfunction of small nociceptive fibers leads to enhanced nerve excitability
causing neuropathic pain. This peripheral denervation can lead to the development of maladaptive plasticity in the central nervous system which also contributes to the generation of neuropathic pain. Central sensitization, which refers to this plasticity and increased activity of peripheral nociceptors, has also been found to be one of the underlying mechanisms of phantom limb pain.
- Purkinje cells are the output cells of the cerebellar cortex. In patients with ASD
cerebellum and limbic system is abnormally structured. There is a reduction in the number or atrophy of Purkinje cells in the cerebellum mostly affecting the posterolateral neocerebellar cortex and adjacent archicerebellar cortex. Basket cells are inhibitory to Purkinje cells and are found in the molecular layer of the cerebellum. Some studies have shown a decrease in the number of basket cells, but most show no change in the number of basket cells in patients with ASD. Stellate cells are inhibitory to Purkinje cells and are found in the outer two-thirds of the molecular layer of the cerebellum. There is no change identified in the number of stellate cells in patients with ASD. Golgi cells are inhibitory cells found in the granular layer of the cerebellum. There is no change identified in the number of Golgi cells in patients with ASD.
- Binswanger's disease - dementia plus 2: HTN or vascular disease
cerebral vascular disease, subcortical dysfunction (neurogenic bladder, rigidity, gait abnormalities), or bilateral MRI/CT abnormalities in periventricular regions; affect lability, dysarthria, dysphagia. Binswanger's disease. This condition is characterized by the 4 D's: dysmnesia (impaired recall that is improved with cuing), dysexecutive syndrome, delay (slowness), and depletion (apathy and amotivation).
- Normal pressure hydrocephalus (NPH) is a potentially reversible chronic disorder that affects elderly patients
characterized by dilation of the cerebral ventricles, as well as a triad of gait apraxia, impaired cognitive function and urinary incontinence.
- This patient has separation anxiety disorder
characterized by excessive fear of separation from the attachment figure, which is developmentally inappropriate. It has been found that oxytocin plays an important role in anxiety regulation and attachment behavior. Lower levels of oxytocin have been implicated in separation anxiety disorder, compared with other anxiety disorders.
- At 4 years of age
children can be expected to draw a square or a cross, throw a ball overhand, hop on 1 foot, use scissors to cut out shapes, and count to 4 pennies. When drawing a person, a 4-year-old will often draw a combined head and body. That is, the 4 limbs will originate from a central circular shape that has a face. This is the combined head and body. A child can be expected to draw a person with a head, neck, hands, and feet beginning at 6 years of age. Most children can ride a tricycle well by 3 years of age.
- Geschwind syndrome - hyperreligiosity
circumstantiality, hypergraphia, hyposexuality, and intense emotional responses; most often temporal lobe epilepsy on the left side - focal seizures with impaired awareness (complex partial seizures)
- OCD:
clomipramine, Fluoxetine fluvoxamine, paroxetine, sertraline,
- Laboratory evaluation for plasma levels for many antipsychotics is available including haloperidol
clozapine, fluphenazine, trifluoperazine, and perphenazine., haldol
- The hallmark triad of Huntington's Disease is movement disturbance
cognitive decline, and psychiatric disturbance. The movement disturbances include chorea and loss of motor control, often manifested initially by clumsiness and incoordination. On structural imaging, the basal ganglia are often reduced in size, especially the caudate nuclei.
- The Stroop Color Word Test is an examination that tests attention
concentration, orientation, and vigilance. In the first trial, the written color name differs from the color ink it is printed in, and the participant must say the written word. In the second trial, the participant must name the ink color instead.
- Latah is a
condition in which an individual is hypersensitive to fright.
- Herpes simplex encephalitis typically presents with a several day history of fever and headache followed by memory loss
confusion, and seizures. The hallmark sign is a focal neurologic deficit suggestive of a structural lesion in the frontal-temporal areas. The EEG is often abnormal, demonstrating periodic sharp-wave complexes from one or both temporal regions on a background of low amplitude activity. PLEDS are often seen. On MRI images, the infection appears as regions of high signal intensity on T2 weighted images and fluid-attenuated inversion recovery (FLAIR) images. CSF should be sent for PCR assay for herpes simplex virus. Treatment is with intravenous acyclovir.
Cognitive-behavioral theory of depression Beck Identifies and tests automatic thoughts
corrects over generalizations; cognitive triad: negative cognition of self, world, and future lead to depression
- Temporal arteritis (aka giant cell arteritis) involves inflammation of the temporal arteries
cranial arteries and often other medium-sized arteries outside of the central nervous system. It primarily occurs in patients older than 55 years of age. Dull and continual pain over one or both temples and accompanying signs of system inflammation (e.g. malaise, muscle and joint pain, low-grade fever, weight loss) are common. In advanced cases, the temporal artery may be tender and indurated. Jaw pain on chewing, or jaw claudication, is considered a very specific sign. Eye pain is not a common sign. The erythrocyte sedimentation rate is greater than 40 mm in over 90% of cases. Temporal artery biopsy shows giant cells and is the gold standard test for diagnosis. If left untreated, temporal arteritis can lead to occlusion of affected arteries and thereby result in blindness and stroke. High-dose steroids are the primary treatment.
- Hwa-Byung -
cultural syndrome similar to somatization disorder; 5% prevalence among middle-aged women in Korea
- Spongiform encephalopathy - CJD most common
death in 6/9 mos, alpha helix to beta sheet protein transformation, carried on chromosome 20.CJD is mostly sporadic (85% of cases), in which the disease appears even though the person has no known risk factors. The disease is characterized by rapidly progressive neurocognitive disorder. At first, patients experience problems with muscular coordination, personality changes including impaired memory, judgment, and thinking, and vision impairment. Those with the disease also may experience insomnia, depression, or unusual sensations, but CJD does not cause fever or other flu-like symptoms. As the illness progresses, mental impairment becomes severe. Patients often develop myoclonus and may eventually go blind. In the later stages of CJD, patients eventually lose the ability to move and speak and enter a coma. Pneumonia and other infections also frequently occur in these patients and can lead to death. The 14-3-3 protein is a marker for classic Creutzfeldt-Jakob disease.
- Thoughts of Psychological debriefing for treatment of PTSD?
debriefing can worsen the PTSD symptoms and none of the major PTSD treatment guidelines recommend psychological debriefing as an initial prevention intervention.
What are Biomarkers in depression
decreased BDNF and Leptin increased inflammation markers CRP and IL-6 and CRH (corticotropin releasing hormone)
- The two TCAs with the lowest anticholinergic activity are?
decreased desipramine and non - nortriptyline.
Object relations theory Klein Tension between true and false self; depression is guilt for previous aggression towards loved object; focused on paranoid/ schizoid and depressive positions
defense mechanisms, analyzed child's play
- Clozapine treatment has caused agranulocytosis
defined as an ANC less than 500/mm3. Agranulocytosis can lead to serious infection and death. Prior to initiating treatment with clozapine, obtain a baseline WBC count and ANC. The ANC must be greater than or equal to 2000/mm3 and the WBC must be greater than or equal to 3500/mm3 for a patient to begin treatment with clozapine. During treatment, patients must have regular monitoring of ANC and WBC. Discontinue clozapine and do not rechallenge if the ANC is less than 1000/mm3 or the WBC is less than 2000/mm3.
- Cyclothymia
defined by low-grade hypomanic and depressive symptoms over two years (at least one year for children and adolescents), not due to a specific due to substance use, and associated with significant distress or impairment. Patients with cyclothymia experience intense and rapid mood fluctuations of opposite polarity leading to interpersonal and behavioral consequences that usually prove to be the most prominent clinical expression of the disorder. This must be present for at least 2 years.
- Positron emission tomography (PET) scans produce images that show concentrations of radioactive compounds injected into patients; this is useful in detecting brain tumors
degenerative diseases, and locating epileptic foci.
- Kubler Ross Death & Dying Sequence:
denial, anger, bargaining, sadness, acceptance
- People with chronic physical illness in childhood may be most prone to what personality disorder?
dependent personality disorder.
- Nutmeg taken in a high enough dose can induce?
depersonalization, derealization, and a sensation of heaviness in the limbs.
- What is Habituation?
describes the process of decreased response to repeated stimuli. This allows the brain tends to retain only information that is important and ignore inconsequential information.
- Tremulousness is an initial stage of alcohol withdrawal
developing after 6 to 8 hours of abstinence. Psychotic and perceptual symptoms such as delusions and hallucinations can occur in 8 to 12 hours. Alcohol withdrawal seizures occur within the first 12 to 24 hours of abstinence, and can occur even while the blood alcohol level remains elevated and the patient remains partially intoxicated. Alcohol withdrawal delirium (delirium tremens) usually occurs within 72 hours, although patients remain at risk for the first week.
- Aicardi syndrome is an X-linked dominant (lethal in males) syndrome characterized by agenesis of the corpus callosum
developmental delay, and infantile spasms. Associated features may include colobomas of the eye or microphthalmia.
- Although multiple neurotransmitters have been implicated in the development of delirium
disturbances of the acetylcholinergic system are most highly correlated with the development of delirium.
- Boundary crossings
do not harm the patient may facilitate the therapeutic process, and may be ethical when engaged in judiciously, whereas boundary violations are harmful or exploitative to the patient and are always unethical.
- What Anti-depressants should be avoided in liver disease?
duloxetine, bupropion, trazodone, tianeptine, iproniazid, nefazodone, phenelzine, imipramine, amitriptyline, and agomelatine are to be avoided in patients with liver disease since they are associated with higher risks of hepatotoxicity.
Experimental humanistic approach Encourage connected
emotional experiences to get out of narrow roles
- Pick cells (distinctive
enlarged vacuolar neurons with argentophilic neuronal inclusions in the cytoplasm bodies) are seen in neurocognitive disorder due to frontotemporal lobar degeneration (as known as frontotemporal neurocognitive disorder).
- A neurological examination and EEG should be performed in patients with antisocial personality disorder
especially those who are aggressive. Abnormal EEG and positive neurological findings might suggest brain damage in childhood. EEG and neurological examinations are also performed to confirm the diagnosis of antisocial personality disorder.
- The following lists the rates of absorption and average half-lives of these agents and their active metabolites in hours:
estazolam rapid, 17 hours; flurazepam, rapid, 100 hours; oxazepam, slow, 8 hours; temazepam, medium, 11 hours triazolam, rapid, 2 hours.
Consider cluster headache in a man with short (15 to 120 minutes)
excruciatingly severe bouts of headache or eye pain associated with ipsilateral autonomic accompaniments with attacks occurring 1 to several times per day.
- Trigeminal
facial, glossopharyngeal and vagus nerves arise from the pharyngeal arches.
- Specific phobias are classified into how many and what types?
five types; -animals (spidersinsects, dogs), -natural environment type (heights, storms, water), -blood injection injury type (needles, invasive medical procedures), -situational type (airplanes, elevators, enclosed spaces) -others that include phobias that do not fit into the previous four types.
- William's syndrome - deletion of 26 genes on chromosome 7; ID
flattened nasal bridge, wide-spaced teeth, prominent lips, starburst irises, very social and ease with strangers, cardiac problems (AS murmur), strabismus
- Social anxiety disorder:
fluvoxamine, Paxil, Sertraline, Venlafaxine
- Individuals with fragile X syndrome also have significant symptoms of autism spectrum disorder
for example, social and communicative difficulties which are very similar to those seen in autism spectrum disorder. In Landau-Kleffner syndrome, generally between the ages of 3 and 7 years, there is a loss (sometimes very rapid) of receptive and expressive language and social skills. This can, therefore, be mistaken for autism spectrum disorder.
- Complex regional pain syndrome occurs after a sprain
fracture or trivial traumatic injury to a limb and is a painful inflammatory condition. This syndrome involves sensory, vasomotor, sudomotor and motor changes. The sensory changes can be hyperesthesia, hyperalgesia, and allodynia. The vasomotor changes involve temperature asymmetry and skin color changes. The sudomotor changes are the presence of edema and sweating changes. Motor dysfunction can also be present. The presence of at least one sign from 3 or 4 of these categories leads to this diagnosis.
- Characteristic features of a cerebellar hemorrhage include inability to walk
gaze palsy without hemiplegia, and absence of unilateral limb paresis. Occasionally, patients may have a single prodromal episode of dizziness or facial numbness. The most common symptom is an inability to stand or walk. Nausea and vomiting usually follow. Dizziness is common and is described as a feeling of insecurity, a drunken feeling, or wavering rather than true rotational vertigo (but vertigo can occur). Occipital headache is also very common. Neurologic exam reveals a combination of a unilateral cerebellar deficit with variable signs of ipsilateral pontine involvement, the latter including gaze palsy, peripheral facial palsy, nystagmus, miosis, decreased corneal reflex, abducens nerve palsy, and skew deviation.
- In neurosyphilis
general paralysis of the insane occurs with widespread parenchymal damage and can present with changes in personality (disinhibition), intellect (memory impairment), and sensorium (delusion of infestation). Tabes dorsalis presents with signs and symptoms from the degeneration of the posterior columns, dorsal roots, and dorsal root ganglia of the spinal cord. These include lancinating (sudden, brief lightning-like stabbing) pains that can affect the limbs or face, which can be seen in this patient. Paresthesias, sensory ataxia, bladder dysfunction, diminished reflexes, and loss of positional, pain, and temperature sensations are also a result of damage to the spinal cord.
- If anisocoria is present and one of the pupils
generally the larger one, reacts poorly to light, the diagnosis can be narrowed to 4 possibilities: third nerve palsy, damage to the ciliary ganglion or the short ciliary nerves (which results in a tonic pupil), damage to the iris due to ischemia, trauma, or an inflammatory process may cause mydriasis, and mydriasis which may be induced by the instillation of a para-sympatholytic drug (e.g., atropine, scopolamine). Unilateral mydriasis may follow the use of transdermal scopolamine to prevent motion sickness, the accidental instillation into the eye of fluids from certain plants (e.g., jimsonweed) that contain belladonna and atropine like alkaloids, and exposure to certain cosmetics and perfumes.
- Polysomnography studies in patients with bipolar disorder have shown what?
greater variability in sleep patterns along with lower sleep efficiency, increased number of arousals, and fragmented sleep. There is a higher density of rapid eye movement sleep in patients with bipolar disorder. There is an inherent instability and blunting of biological rhythms in these patients. Reduced sleep efficiency and rhythm robustness in patients is associated with abnormal dorsolateral prefrontal cortical response. There is an increased sleep latency in patients with bipolar disorder. There is an increased duration of sleep latency in patients with bipolar disorder. There is a reduced daily activity in patients with bipolar disorder.
- Hypercalcemia - stones
groans (bone pain), psych overtones (mental slowing with attention and memory problems)
- GCA occurs in patients of either sex or age greater than 55 years. This vasculitis carries a risk of permanent visual loss in 20% to 50% of patients if untreated. Associated systemic complaints include polymyalgia rheumatica
headache (in 40% to 90%), weight loss (16% to 76%), scalp tenderness (28% to 91%), anorexia (14% to 69%), fever, leg claudication (2% to 43%), and jaw claudication (4% to 67%) (uncommon, but relatively specific). Although jaw claudication is considered relatively specific for GCA, other etiologies include Wegener's granulomatosis, polyarteritis nodosa, primary systemic amyloidosis, hairy cell leukemia, Langerhans cell histiocytosis, sarcoidosis, parotid duct obstruction, and atherosclerosis of external carotid artery. Any new headache in a patient 55 years of age or older should be considered due to GCA unless proven otherwise. There is nothing very specific about the headaches with GCA. Headaches may be unilateral or bilateral and are often temporal but may involve any or all aspects of cranium. Headaches also may be of gradual or explosive onset. The quality of pain is seldom throbbing and more often dull or boring with superimposed episodic icepick-like, lancinating pain. The headaches are often worse at night and often aggravated by exposure to cold. Scalp tenderness is often present. Visual loss in GCA is usually due to anterior ischemic optic neuropathy (AION) (5% of patients with AION have GCA). There may also be transient visual loss (due to transient ischemia to the retina, choroid, optic nerve, or brain), central retinal artery occlusion (5% to 10% of central retinal artery occlusion (CRAO) patients have GCA; always consider GCA as cause of CRAO if no embolus visible), branch retinal artery occlusion (less frequent), choroidal ischemia ("triangular syndrome"), or cotton wool spots. If one eye losses vision, without treatment, the second eye will be affected in 75% (simultaneous bilateral involvement highly suggestive of GCA). If the fellow eye is affected: 1/3 will occur in 24 hours, another 1/3 within 1 weeks, and most of the remainder within 4 weeks. The risk of visual loss after 6 months is low. Corticosteroids are used to protect the other eye but rarely improve vision. Do not use alternate day steroids.
- Indomethacin-responsive headaches include paroxysmal hemicranias (a subtype of trigeminal-autonomic cephalgias)
hemicrania continua, valsalva-induced headaches, and primary stabbing headache.
- Proximal neuropathy tends to cause pain in the thighs
hips, or buttocks, and can lead to weakness in the legs. Finally, focal neuropathy results in the sudden weakness of 1 nerve or a group of nerves, causing muscle weakness or pain. Any nerve in the body can be affected.
- Edwards syndrome is Trisomy 18; it is the second most common autosomal trisomy observed in live births. The clinical features include hypertonia
horseshoe kidney, smallmouth, micrognathia, pointy ears, short sternum, prominent occiput and flexed fingers with the index finger overlapping the third finger and the fifth finger overlapping the fourth.
- Kleine-Levin syndrome - male adolescents with episodic excessive daytime sleepiness
hyperphagia, hypersexuality; possible hypothalamus damage
- Remember that the electrolyte abnormalities associated with anorexia nervosa all tend to be; hypo:-hypochloremia
hyponatremia, hypomagnesemia, hypokalemia, hypophosphatemia, and sometimes hypocalcemia. Bicarbonate levels increase due to the alkalosis associated with vomiting of gastric fluid.
- The most common type of "popper" is amyl nitrite which can cause vasodilation
hypotension, and tachycardia.
- Metencephalon gives rise to the pons and cerebellum. Diencephalon gives rise to the thalamus
hypothalamus, and epithalamus. Mesencephalon gives rise to the midbrain. Myelencephalon gives rise to the medulla oblongata. Telencephalon gives rise to the cerebral hemispheres.
- Hypnic Headache. Short-lasting headache syndromes with no autonomic features include the following entities: trigeminal neuralgia
idiopathic stabbing HA, Valsalva maneuver headache, cough headache, benign exertional headache, headache associated with sexual activity, and hypnic headache. Hypnic headaches are benign, recurrent headaches that occur exclusively during sleep in individuals usually older than 50 years of age. Consider cluster headache in a man with short (15 to 120 minutes), excruciatingly severe bouts of headache or eye pain associated with ipsilateral autonomic accompaniments with attacks occurring 1 to several times per day.
- Short-lasting headache syndromes with no autonomic features include the following entities: trigeminal neuralgia
idiopathic stabbing HA, Valsalva maneuver headache, cough headache, benign exertional headache, headache associated with sexual activity, and hypnic headache. Short-lasting headache syndromes with autonomic features (trigeminal autonomic cephalgias) include the following entities: cluster headache, paroxysmal hemicranias (chronic and episodic), SUNCT syndrome, and short-lasting unilateral neuralgiform headache attacks with cranial autonomic symptoms (SUNA syndrome).
- According to the guidelines for trauma what is recommended for mild symptoms?
if mild symptoms are present for less than 4 weeks, then "wait and watch" is recommended.
- Williams syndrome is a genetic disorder where the child tends to also have poor language skills; however
in Williams syndrome, the child is very friendly and often trusts strangers.
- The first line treatment for cluster headaches is with a calcium channel blocker
in particular verapamil at higher doses. Sumatriptan and naratriptan are both triptans and are indicated for acute or abortive treatment. Dihydroergotamine is indicated as first line abortive treatment and as second line preventive treatment. Supplemental oxygen is also first-line abortive treatment.
- The first myelination is seen as early as the 16th week of gestation
in the column of Burdach, but only really takes off from the 24th week. It does not reach maturity until 2 years old.
- The ventrolateral prefrontal cortex plays a role in suppressing maladaptive emotional responses. Impairment of ventrolateral prefrontal cortex in bipolar disorder leads to disinhibited and socially inappropriate behaviors which are frequently observed in patients during mania. There is a decreased activity in the dorsolateral prefrontal cortex in patients with bipolar disorder and is associated with impaired working memory
inability to maintain attention, and compromised executive function. The amygdala is associated with the evaluation and interpretation of emotion of an ambiguous stimulus. Patients with bipolar disorder have inappropriate responses to change in surroundings and have difficulty in the interpretation of emotional meaning of facial expressions. Dorsal anterior cingulate cortex or mid-cingulate cortex is associated with cognition, and along with ventral anterior cingulate cortex, plays a part in cognitive-emotional integration and monitoring of behavior. Ventral anterior cingulate cortex comprises of the subgenual and subcallosal or pregenual anterior cingulate cortices involved with regulation of emotions, and along with dorsal anterior cingulate cortex, plays a part in cognitive-emotional integration and monitoring of behavior.
- The Dissociative Experiences Scale (DES) is a common tool to assess the severity of a wide variety of dissociative experiences
including identity alteration, depersonalization-derealization and amnesia. It is a 28-question self-report questionnaire.
- The thalamus is the generator and modulator of EEG rhythms
including the alpha rhythm. Childhood absence epilepsy is believed to originate from the thalamus.
- Perifornical nuclei are responsible for hunger
increased blood pressure and rage.
- MOA of Valproic acid, what it treats, ways it affects other medications?
increases Na+ channel inactivation and increases GABA concentration by inhibiting GABA transaminase. It is used in the treatment of simple and complex, tonic-clonic, absence and myoclonic seizures, migraine prophylaxis, acute manic/mixed episodes in adults and relapse prevention in bipolar disorder. Valproic acid increases serum concentrations of different drugs including lamotrigine, carbamazepine, ethosuximide, phenytoin and phenobarbital by inhibiting their hepatic metabolism. Clinically significant rise in the concentration of lamotrigine occur in blood when it is co-administered with valproic acid due to slow hepatic metabolism.
- Eszopiclone a non-benzodiazepine hypnotic is indicated?
indicated for in the treatment of insomnia characterized by difficulty with sleep onset and sleep maintenance.
- Doxepin is a tricyclic antidepressant indicated in sleep for?
indicated for the treatment of insomnia characterized by difficulty with sleep maintenance.
- When is Zaleplon indicated? is a non-benzodiazepine hypnotic
indicated for treatment of insomnia characterized by difficulty with sleep onset only, but not with sleep maintenance because of its short half-life.
- Estazolam is a benzodiazepine indicated?
indicated in the treatment of insomnia characterized by difficulty with sleep onset and sleep maintenance.
- Suvorexant is an orexin receptor antagonist indicated?
indicated treatment of insomnia characterized by difficulties with sleep onset and/or sleep maintenance.
- Duchenne muscular dystrophy (DMD) is characterized by extremely high serum creatine kinase (CK) levels. Although high CK levels are non-specific
indicating the likely destruction of muscle tissue from whatever cause, they can distinguish a muscular etiology from CNS and peripheral nerve etiologies. Because of its wide availability and rapid results, CK levels are usually obtained early in the diagnostic process. The definitive test for DMD is genetic testing.
- Between the ages of 10 and 12 months
infants develop social referencing wherein they look to a caregiver's response to a new situation to determine how they should respond to it.
- Disulfiram works by?
inhibiting dopamine beta hydroxylase which converts dopamine to norepinephrine, and can thereby increase dopaminergic neurotransmission in the brain. Excessive dopaminergic neurotransmission in the mesolimbic pathway is thought to lead to positive psychotic symptoms.
- Generalized anxiety disorder dx?
intense worry about various events for a number of days, but for at least 6 months and is associated with the presence of at least 3 of the symptoms from restlessnesseasily fatigued, difficulty concentrating, irritability, muscle tension, and sleep disturbances.
- Aripiprazole and risperidone have both received an FDA indication for the treatment of?
irritability associated with autism spectrum disorder.
- Schilder disease
is a rare, poorly understood demylinating disease that in some ways resembles multiple sclerosis. A typical brain MRI finding is T2 signal hyperintensity occupying much of the semioval center bilaterally.
- Rett syndrome occurs in girls between 6 and 18 months of age
is characterized by loss of milestones and decrease in head circumference. It is believed to be caused by mutation in the MECP2 gene.
- Treatment-emergent weight-gain is defined as?
is defined as a 7% increase from baseline
- What is Memory sensitization?
is the process through which repeated stimuli leads to increased response to repeated stimuli.
- Lithium-induced nephrogenic diabetes insipidus is caused by?
is usually a self-limiting condition in which there is a down-regulation of channel aquaporin-2 in the luminal wall of the cell's surface leading to decreased urine osmolality and increased urine volume (polyuria).
- A newborn may intently look at the high contrast objects because her vision is only limited to black and white and can hardly focus 10-12 inches away from her eyes. Color vision is not well-developed when a child is born. However
it gets developed by the end of the third month of age, and then, the child can differentiate between different wavelengths of light. It occurs due to the development of various subclasses of cones. Similarly, a child cannot accommodate his vision when newly born, but by the age of 3 months, a child will be able to follow an object, and by the age of 6 months, a child can accommodate his vision.
- Life-threatening dysrhythmias such as ventricular fibrillation or tachycardia can occur in patients with hydrocarbon misuse. The exact mechanism by which these substances cause ventricular dysrhythmias is not fully understood. However
it is suggested that certain volatile substances sensitize the myocardium to catecholamines.
- Dantrolene is a skeletal muscle relaxant
it may provide some symptomatic relief, but can exacerbate serotonin toxicity.
- If a patient has an inadequate response to an antipsychotic after six weeks what is the next step?
it should be switched.
- Although spinal stenosis may affect multiple nerve roots
its course is chronic and it rarely produces fixed weakness.
- The Leiter International Performance Scale is a nonverbal
language free test is used as a measure of intelligence in individuals who have difficulty in responding to traditional tests.
- The Leiter International Performance Scale is a nonverbal
language free test used as a measure of intelligence in individuals.
- Opitz G/BBB syndrome is also a genetic disease described by hypertelorism
laryngotracheal-esophageal abnormalities (with a resultant hoarse cry, recurrent stridor, swallowing difficulties), oral clefts, genitor-urinary anomalies (such as hypospadias or splayed labia majora), imperforate anus, congenital heart defects, and developmental delay.
- Child abuse is a predictive factor for?
later onset of panic disorder.
- Ablation of the medial temporal lobe causes damage to the bilateral amygdalas as well
leading to Kluver-Bucy syndrome, in which the patient develops excessive tameness as well as hyperphagia, hyperorality and hypersexuality. Ablation of the anterior cingulate gyri and subcallosal gyri leads to increase in viciousness and rage as there is no longer any inhibitory effect on the rage centers. Ablation of the anterior cingulate gyri and subcallosal gyri leads to increase in viciousness and rage as there is no longer any inhibitory effect on the rage centers. Ablation of the hippocampus would result in anterograde amnesia Prefrontal association area helps to plan complex motor movements and sequences of motor movements by working in conjunction with the motor cortex. It is also responsible for higher intellectual functions.
- Spinal cord neoplasms frequently affect the pyramidal tract
leading to areflexia and extensor plantar responses
- Epidural hematomas are usually caused by trauma and the middle meningeal artery tends to be involved in the formation of this hematoma. Blood collects rapidly in the epidural space and starts compressing on the underlying parenchyma
leading to neurological symptoms like headache, nausea, vomiting and focal neurologic deficits. It classically presents with a lucid interval which then leads to deterioration. In an acute subdural hematoma, the blood collects between the dura and brain parenchyma and most patients end up having a severe initial GCS score (8 or less), which was not the case here. The traumatic incident also tends to be moderate to severe rather than mild. A deep intracerebral hemorrhage is most likely caused by hypertension whereas a lobar hemorrhage comes about spontaneously. The clinical presentation of an intracerebral hemorrhage is dependent on the area involved e.g. a putaminal hemorrhage would result in contralateral hemiplegia and conjugate deviation of the eyes.
- Diffusion-tensor imaging (DTI) is a complex form of diffusion-weighted imaging that can be used to assess the integrity of white matter fibers and document the extent of microstructural damage after traumatic brain injuries. The FLAIR (fluid-attenuated inversion recovery) MRI is particularly useful for visualizing lesions located near CSF compartments
like multiple sclerosis and subarachnoid hemorrhages.
Dx criteria for MDD?
loss of interest/pleasure or depressed mood + (4) sleep changes, guilt, concentration, unwanted loss wt/appetite, fatigue, suicidal thoughts most of the time, and the persistence of symptoms for almost every day for more than 2 weeks. - DSM-5 DX: (5 Sx) loss of interest in pleasurable activities, insomnia, fatigue, suicidal thoughts, and depressed mood most of the time, and the persistence of symptoms for almost every day for more than 2 weeks. - Short-duration depressive episode, is an "other specified depressive disorder". This is characterized by 4-13 days of depressed mood with the presence of any one of the 8 other symptoms required for the diagnosis of a major depressive episode. - Dysthymia is characterized by low mood for most of the days on most days for at least "Dy"-two years. It is diagnosed when there are at least "Dy" two symptoms present from the following list for at least two years: poor appetite or overeating, insomnia or hypersomnia, fatigue and low energy, low self-esteem, feelings of hopelessness and poor concentration. - DSM V indicates that onset within 4 weeks of delivery must occur for the diagnosis of major depressive disorder with postpartum onset.
- Tourette's disorder - deficits in frontal cortex (low creatine on right
low myoinositol on left), caudate, putamen (reduced choline, n-acetylaspartate), and thalamus
- There is a decrease in somatomedin C
luteinizing hormone (LH) and FSH levels in patients with anorexia nervosa similar to prepubertal levels. CRH and cortisol typically increase.
- Methyldopa (antihypertensive) may induce what?
may induce or worsen Depression.
- If patient undergoes hepatic shunt, What should happen to medications (most SSRIs and Antipsychotics are metabolized by liver; not Paxil, Luvox, (es)citalopram or paliperidone)?
medication dose should be decreased when a patient undergoes a hepatic shunt.
- Chlorpromazine
metformin, bupropion, olfloxacin, dimethylamylamine, labetalol, promethazine, trazodone, and fenofibrate are drugs that can cause false positives in commercial urine drug-screen amphetamine immunoassays.
- Gradient echo (GRE) MRI has been used to detect iron deposition in organs
microbleeds, as well as early stages of degenerative disease. Magnetic resonance spectroscopy (MRS) can be used to detect cellular metabolites and evaluate CNS disorders, including brain tumors and neurodegenerative disorders.
- Downbeat nystagmus may occur with cervicomedullary junction disease
midline medullary lesions, posterior midline cerebellar lesions, or diffuse cerebellar disease. Most responsible lesions affect the vestibulocerebellum (flocculus, paraflocculus, nodulus, and uvula) and the underlying medulla. Deficient drive by the posterior semicircular canals, whose central projections cross in the floor of the fourth ventricle, has been postulated as an explanation for downbeat nystagmus. Downbeat nystagmus is common with cervical-medullary junction processes (e.g., Chiari malformation). Nystagmus, or vertigo induced by the coughing or the Valsalva maneuver, may occur with Chiari malformation or perilymph fistulas. Benign paroxysmal positional vertigo and Meniere's disease do not cause nystagmus.
Self-psychological theory Kohut Deficits/disintegration of self
mirroring and idealization; supportive, empathic, validating, and introspective approach to NPD
What are Atypical features of depression?
mood reactivity + 2/4: significant weight gain/increase in appetite, hypersomnia, leaden paralysis, interpersonal rejection sensitivity (screen for bipolar; more common in women)
- Lewy bodies are composed of alpha synuclein
neurofilament, and ubiquitin.
- Wilson's disease - AR disease with mutation in ATP7B gene on chromosome 3; memory problems
neuropsych sx, jaundice, tremor, Kayser-Fleischer rings, low serum copper, low ceruloplasmin, high urine copper; Tx: penicillimine and zinc
- Due to cocaine's mechanism as a vasoconstrictor
non-hemorrhagic infarcts are common
- Tyrosine is the correct answer as it is the amino acid precursor of all the catecholamine neurotransmitters including dopamine
norepinephrine, and epinephrine.
- Tourette's disorder requires multiple motor and one or more vocal tics present at some time during the illness
not necessarily concurrently, persisting for more than one year, with onset younger than 18 years of age. Persistent (chronic) motor or vocal tic disorder differs in requiring single or multiple motor or vocal tics, with the same age and time parameters as Tourette's. Provisional tic disorder is the presence of a single tic or multiple motor and/or vocal tics, present for less than 1 year since the first tic onset.
- Chronic alcohol consumption affects brain receptors
notably GABA receptors (decreases) on inhibitory neurons. In addition, chronic alcohol affects NMDA receptors (via glutamate) on excitatory neurons (increases receptors).
- Illness anxiety disorder includes a preoccupation with fears of having or acquiring a serious disease based on the person's misinterpretation of bodily symptoms. The preoccupation exists despite receiving a medical evaluation and reassurance. The preoccupation is not attributed to delusions and is not restricted to concern about appearance. The preoccupation causes significant distress and impairment in social
occupational, and other areas of functioning. The duration is at least 6 months.
- Age of minors can consent to testing & tx for STIs
of at least 12-years of age, to consent to sexually transmitted infections (STI) diagnosis and treatment without parental/ guardian consent. However, the age differs from state to state. For instance, it is 12 years or older for California, but 14 years or older for Idaho.
- Alcoholic ketoacidosis is associated with decreased food intake
often because of abdominal pain accompanied by nausea and vomiting. This absence of carbohydrate ingestion leads to glycogen depletion in the liver with a compensatory increase in glucagon, which promotes lipolysis to form free fatty acids as an energy substrate.
- Wernicke-Korsakoff syndrome: a condition of gait ataxia
ophthalmoplegia, and memory impairment that occurs in patients with chronic thiamine deficiency. It is heavily associated with severe chronic alcohol use. Brain MRI revealing lesions of the medial thalamic and periaqueductal areas will be seen.
- Devic's disease
or neuromyelitis optica (NMO), is an autoimmune central nervous system demyelinating disorder which preferentially affects the optic nerves and spinal cord. Diagnostic criteria include optic neuritis and transverse myelitis and the presence of at least 2 out of the 3 of the following: 1) antibody to NMO (aquaporin-4) IgG; 2) brain MRI not consistent with multiple sclerosis; or 3) longitudinally extensive (greater than 3 vertebral segments) spinal cord lesion.
- Vitamin B3
or niacin, deficiency is the most common cause of pellagra. In addition to diarrhea, dermatitis, and dementia, it may involve photosensitivity, glossitis, ataxia, and dilated cardiomyopathy. Left untreated, death from the disease will occur within 4 or 5 years.
- Anterior hypothalamus - cooling
parasympathetic
- The CDC requires all patients currently undergoing treatment for tuberculosis be screened for HIV infection. In addition
parental consent for HIV screening is only required if the patient is younger than 13.
- Meperidine withdrawal begins within 4 to 5 hours
peaks in 8 to 12 hours, and is over by 4 to 5 days. Morphine withdrawal begins in 6 to 8 hours, peaks in the second or third day, and is complete by 7 to 10 days. Methadone withdrawal begins 1 to 3 days after the last dose, and ends in 10 to 14 days. Buprenorphine withdrawal has been observed to peak at about 5 days after the last dose and is generally mild, although symptoms may persist for weeks.
- Fabry disease - X-linked lysosomal deficiency; GI symptoms
peripheral neuropathy, angiokeratomas, cardiac/renal disease
- Long term use of disulfiram has been associated with?
peripheral neuropathy.
- What does Propranolol leads to in peripheral vasculature?
peripheral vasoconstriction.
- The Wisconsin Card Sorting Test is a test that assesses the problem-solving ability of an individual by measuring their attention
perseverance, intellectual flexibility, abstract thinking, concept formation, and set shifting.
- Chronic fatigue syndrome is unexplained
persistent, or relapsing chronic fatigue lasting 6 or more consecutive months that is of new or definite onset; is not the result of ongoing exertion; is not substantially relieved by rest; and results in substantial reduction in previous levels of occupational, education, social, or personal activities, and 4 or more of the following symptoms occurring concurrently: (1) impairment of short-term memory or concentration; (2) sore throat; (3) tender cervical or axillary lymph nodes; (4) muscle pain, or multi-joint pain; (5) headaches; (6) unrefreshing sleep; and (7) post exertional malaise.
- The phenylpiperidine series of opioids increase risk of?
pethidine (meperidine), tramadol, methadone, dextromethorphan, and propoxyphene, appear to be weak serotonin reuptake inhibitors and have all been involved in serotonin toxicity reactions with MAOIs (including some fatalities).
- Benadryl is an antihistamine made of diphenhydramine and its overdose is treated by giving what?
physostigmine
- γ-aminobutyric acid A receptor subunit α2 (GABRα2) is a marker of GABAnergic neurotransmission during neurodevelopment and is found in high levels in lateral cerebellum of patients with schizophrenia. Synaptophysin is a marker of synaptogenesis and axonal growth during neurodevelopment and is found in low levels in prefrontal cortex and hippocampus of patients with schizophrenia. Brain-derived neurotrophic factor (BDNF) is a marker of synaptic connections during neurodevelopment and is found in low levels in hippocampus of patients with schizophrenia. Glutamate Decarboxylase 1 (GAD1) is a marker of synthesis of glutamate and GABA during neurodevelopment and is found in low levels in lateral cerebellum of patients with schizophrenia. Glutamate decarboxylase 67 (GAD67) is a marker of glutamatergic neurotransmission during neurodevelopment and is found in low levels in lateral cerebellum
prefrontal cortex, and temporal neocortex of patients with schizophrenia.
- Sedating antipsychotics raise the risk of embolic phenomena during?
pregnancy and postpartum. embolic stroke occurs when a blood clot that forms elsewhere in the body breaks loose and travels to the brain via the bloodstream. When the clot lodges in an artery and blocks the flow of blood, this causes a stroke. This is a type of ischemic stroke.
- Secondary causes of RLS include iron deficiency anemia
pregnancy, uremia, or peripheral neuropathy. Patients with a ferritin of less than 50 ng/mL may experience improvement of RLS symptoms with iron replacement therapy, typically with oral ferrous sulfate.
- Development of psychiatric illness post-TBI involves a host of factors from premorbid biological disposition
premorbid psychological factors, post-injury biological changes, and one's psychosocial and environmental context. Pre-morbid psychiatric history has been demonstrated to be a significant risk factor for multiple psychiatric sequelae following TBI.
- Several gene mutations have been identified as risk factors for the development of Alzheimer's disease (AD). Autosomal dominant mutations in the Presenilin1 gene (PSEN1) results in mutations in the presenilin protein
presenilin 1 (PS1). This protein is a part of the γ-secretase complex, which is responsible for proteolytic processing of the amyloid precursor protein (APP). Abnormal processing of APP results in accumulation of amyloid-beta peptides (Aβ), which cause neural toxicity and contribute to the pathology seen in Alzheimer's disease. Familial forms of early onset AD are associated with abnormal presenilin. Apolipoprotein E epsilon 4 (APOEε4) allele is the primary genetic risk factor for the development of sporadic cases of AD. Each allele is associated with increased risk of deposition of amyloid-β and earlier age of onset. On the other hand, the apolipoprotein E epsilon 2 (APOE ε2) allele seems to be protective against AD, associated with delayed onset of the disease and fewer plaques.
- Dhat is a
psychological disorder which is associated with fear of semen in urine. It is commonly observed in India and patients fear they are becoming infertile.
- Basilar migraine - headache
quadriplegia, stupor, psychosis, blindness, coma
- Acamprosate is more useful in patients who are already abstinent and want to remain abstinent
rather than for patients who are still actively drinking.
Consider trigeminal neuralgia in patients without autonomic features with paroxysms of severe but short-lasting (seconds) paroxysms of pain in a distribution of one of the branches of the trigeminal nerve. Hypnic headaches are benign
recurrent headaches that occur exclusively during sleep in individuals usually older than 50 years of age.
- Panic disorder dx?
recurrent unexpected panic attacks with intense fear that reaches peak within minutes and is associated with 4 of the symptoms from palpitations, sweating, shaking, shortness of breath, choking, nausea, abdominal pain, feeling dizzy or light-headed, chills or heat sensation, paraesthesias, derealization, fear of losing control and fear of dying.
How does Pregabalin and gabapentin work?
reduced Ca2+ channel current by targeting the α2δ accessory subunit of calcium channels. α2δ ligands' voltage-activated Ca2+ channel subunit, is a binding site of gabapentinoids used to treat neuropathic pain and epilepsy. However, it is unclear how α2δ-1 contributes to neuropathic pain and gabapentinoid actions. It also plays an unknown roll in the mechanism of action in controlling symptoms of RLS. As a class, they bind with high affinity to the α2δ-1 subunit of voltage-gated calcium channels, ultimately resulting in inhibition of calcium entry and normalization of neurotransmitter release, including glutamate
- MOA of Gabapentin?
reduced Ca2+ channel current by targeting the α2δ accessory subunit of calcium channels. α2δ ligands' voltage-activated Ca2+ channel subunit, is a binding site of gabapentinoids used to treat neuropathic pain and epilepsy. However, it is unclear how α2δ-1 contributes to neuropathic pain and gabapentinoid actions. It also plays an unknown roll in the mechanism of action in controlling symptoms of RLS. As a class, they bind with high affinity to the α2δ-1 subunit of voltage-gated calcium channels, ultimately resulting in inhibition of calcium entry and normalization of neurotransmitter release, including glutamate Gabapentin does not directly impact GABA-A receptors, which are GABA-gated chloride channels
Type I error
rejecting the null hypothesis and saying there is a treatment effect when the treatment effect does not exist or is incorrect.
- Most criteria for diagnosis of brain death requires a body temperature of what?
require a temperature of at least 32.2 degrees C (90 degrees F). Some combination of Technetium scan. EEG, and cerebral angiogram are obtained in almost all cases although they are considered optional and not required to formally establish brain death.
- A fiduciary obligation
requires a physician to act in the best interest of their patient rather than themselves.
- Alcohol withdrawal delirium is caused by decreased GABA and NMDA receptor inhibition
resulting in delirium and sympathetic overdrive.
- Damage to the thalamic areas can result in what type of amnesia?
retrograde amnesia
- HD - AD on chromosome 4 with CAG repeat; overproduction of a synaptic vesicle-associated protein (huntingtin) triggers an NMDA receptor-mediated excitotoxicity resulting in degeneration of the neostriatum (caudate nucleus and putamen); Tx: tetrabenazine (VMATi)
risperidone
Transactional theory of stress and coping Lazarus & Folkman Primary appraisal: asks if stressor is a threat
secondary appraisal evaluates coping skills
- Adrenoleukodystrophy - XL disorder with accumulation of LCFAs affecting myelin; vision and hearing impairment
seizures, weakness, hyperactivity
- The migraine aura is a complex of neurologic symptoms that may precede or accompany the headache phase or may occur in isolation. It usually develops over 5 to 20 minutes and lasts less than 60 minutes. The aura can be visual
sensory, motor, or any combination of these. The most characteristic visual aura of migraine is a scintillating scotoma (occurring in about 64% of cases), beginning as a hazy spot from the center of a visual hemifield followed by shimmering light of different patterns expanding peripherally to involve a greater part of the hemifield with scotoma.
- Pain fibers: alpha-beta largest and most heavily myelinated; alpha-delta fibers thinly myelinated and transmit fast
sharp first pain, associated with acupuncture and TENS units; C-fibers transmit delayed and dull second pain related to chronic pain, associated with sensitization and NMDA-amplification
- Noonan syndrome is a multisystem genetic disorder characterized by congenital heart defects
skeletal malformation, small stature facial dysmorphism, and mild intellectual disability.
- Fabry's disease is an X-Linked lysosomal storage disease that causes cardiomyopathy
skin lesions and renal failure.
- Joubert syndrome is characterized by agenesis or hypoplasia of the cerebellar vermis and a malformed brain stem. The most common features of the disorder include ataxia
sleep apnea, abnormal eye and tongue movements, and hypotonia.
- Eszopiclone is the only medication listed that is FDA approved for?
sleep initiation and maintenance, having a half-life of about 6 hours.
- Pons - arousal
sleep-wake cycles; reflex control of respiratory system, eye movement, coordination if eyes and head, corneal reflex
- The most frequently occurring primary parasomnias are disorders of arousal and REM sleep disorders. Disorders of arousal include confusional arousals
sleepwalking, and sleep terrors, all of which are characterized by partial arousals from NREM sleep. REM sleep parasomnias include nightmares, which are characterized by frightening dreams and autonomic arousal, and REM behavior disorder, which is characterized by the absence of muscle atonia normally present during REM sleep. Behaviors associated with primary parasomnias may lead to injury of the patient or bed partner. Several secondary parasomnias, such as sleep-related expiratory groaning, or esophageal spasm, have also been studied.
- Hippuric acid can be found in the urine of those using what substance?
sniffing glue.
- Histrionic Personality Disorder is frequently associated with?
somatic symptom disorders.
- One would expect to see the classic Lewy bodies (single large
spherical, brightly eosinophilic inclusion bodies within neuronal cytoplasm) on neuropathologic examination of Lewy body disease.
ARR
subtract %
- REM sleep behavior disorder (RBD) is often associated with "synucleinopathies"
such as Parkinson's disease or Neurocognitive Disorder with Lewy Bodies.
- Pernicious anemia is the most common cause of vitamin B12 deficiency. This can manifest with non-specific symptoms of anemia
such as fatigue and cognitive impairment, as well as glossitis, where the inflamed tongue is enlarged, sore and 'beefy' red. Neurological manifestations result from defective myelin synthesis and include numbness and paresthesia's in the hands and feet, loss of vibration and position sense, diffuse hyperreflexia, absent Achilles reflexes, and a progressive spastic weakness. Subacute combined degeneration of the spinal cord is the classic neurological finding in patients with vitamin B12 deficiency. Cognitive changes including irritability, forgetfulness, severe dementia and even psychosis. Behavioral changes and optic atrophy may also occur.
- Smaller brain lesions
such as meningiomas, are not contraindications for ECT and can be pretreated with dexamethasone to reduce the risk for edema and brain herniation.
- Positron emission tomography (PET) is an imaging technique that allows the examination of active biological processes
such as regional blood flow and glucose metabolism. This can help differentiate FTD from Alzheimer's dementia.
- After stopping the medication
supportive measures should be taken first for the treatment of neuroleptic malignant syndrome, including fluids, cooling, and cardiac support.
- The history of subacute to chronic
symmetrical, distal more than proximal sensory loss weakness with reduced deep tendon reflexes should raise the possibility of an inflammatory polyradiculopathy.
- Posterior hypothalamus - heating
sympathetic
- Both morning glory seeds and catnip can mimic intoxication of?
taken in sufficient quantity, can mimic the intoxication produced by LSD.
- Valbenazine and deutetrabenazine are FDA-approved for the treatment of what?
tardive dyskinesia. Tardive dyskinesia - onset: Months to years Involuntary movements of the mouth and tongue, limbs, face, and respiratory muscles caused by chronic use of antipsychotic drugsRepetitive chewing and lip smackingChoreic movements -Discontinue the causative drug if possible after slowly tapering the dose. [6]Switch to an SGA (associated with fewer EPS, especially clozapine and quetiapine). --Treatment with anticholinergics or a sudden reduction in the dose of antipsychotics may initially worsen the condition. [17] --Start vesicular monoamine transporter-2 (VMAT) inhibitors for moderate or severe symptoms. Valbenazine, Deutetrabenazine, Tetrabenazine ADAPT: Extrapyramidal symptoms include Acute Dystonia, Akathisia, Parkinsonism, and Tardive dyskinesia. Pathophysiology: Inhibition of the nigrostriatal dopaminergic pathways results in EPS. ; First-generation high-potency antipsychotics: D2antagonism → more EPS Second-generation antipsychotics: weaker D2antagonism → fewer EPS
- RNA polymerase II transcribes mRNA by binding to a complex of regulatory proteins called transcription factors to the regulatory region of the gene
termed as the promoter. RNA polymerase I transcribes DNA to synthesize rRNA. RNA polymerase III transcribes DNA to synthesize ribosomal 5SrRNA, tRNA and other small RNAs. Primase synthesizes short stretches of RNA that are complementary and antiparallel to the DNA template. DNA polymerase III causes elongation of the DNA strand by adding deoxyribonucleotides.
Correlation Coefficient
the Strength of the relationship between two study variables
- In patients with PTSD
the anterior cingulate cortex (ACC) is reduced in volume as it is involved in the extinction of conditioned fear, which is impaired. Reduced hippocampal volume is considered a hallmark feature of posttraumatic stress disorder. Prolonged exposure to stress damages the hippocampus and reduction in the volume is seen, which is associated with the severity of the trauma and impairment of memory. In patients with PTSD, there is an increase in the activity of amygdala as it is involved in mediating stress response, emotional processing, and acquisition of fear responses. There is reduced activation of the medial prefrontal cortex (PFC) in patients with PTSD in response to stimuli and it is associated with the severity of the disease. There is a reduction in the volume of frontal cortex in patients with PTSD that leads to dysregulation of executive functions.
- Facilitation helps with the process of sensitization. The more a memory is facilitated
the better it is remembered.
- With Rett syndrome
the child develops normally for the first 5 months (or so) of his/her life. Then, sometime between 6 months and 4 years of age, the child loses social and communication skills. The child is often left with "profoundly impaired expressive and receptive language." In tuberous sclerosis the first clue in an infant is often the presence of seizures, delayed development or white patches on the skin. Between 25% and 61% of affected individuals meet the diagnostic criteria for autism spectrum disorder. In terms of fragile X syndrome, 3% to 5 % of children who have been diagnosed with autism spectrum disorder, in fact, have the FMRI mutation which is the characteristic of fragile X.
- In utero exposure to valproic acid is now a known risk factor for.
the development of nural tube defects (spinabifida), autism spectrum disorder, cleft palate
- In order to diagnose a patient with disruptive mood dysregulation disorder (DMDD) the symptoms must develop before the age of 10 years but cannot apply to children below the age of 6 years. Furthermore
the diagnosis requires that there are severe recurrent outbursts which are manifest verbally and/or behaviorally to a stimulus which does not warrant such an extreme response and occur, on an average, three or more times a week. These outbursts should be such that they severely disrupt life either at home or school and mildly or moderately affect life in the second setting (home or school). Additionally, a persistent irritable mood remains in between outbursts. These episodes must be present for at least a duration of one year.
- Topiramate is cleared by what?
the kidney.
- Dopamine β-hydroxylase converts dopamine to norepinephrine. It is found in low levels in the plasma of patients with conduct disorder as decreased noradrenergic functioning is implicated in the pathophysiology. Acetylcholinesterase breaks down acetylcholine into acetate and choline. Acetylcholine is not implicated in the pathophysiology of conduct disorder; therefore
the levels of acetylcholinesterase are not affected. Glutamate decarboxylase in involved in the synthesis of GABA from glutamate. It is not implicated in the pathophysiology of conduct disorder; therefore, the levels of glutamate decarboxylase are not affected. Glutamate dehydrogenase is involved in the synthesis of glutamate from glutamine. It is not implicated in the pathophysiology of conduct disorder; therefore, the levels of glutamate dehydrogenase are not affected. Catechol-O-methyltransferase (COMT) is involved in the breakdown of dopamine, norepinephrine, and epinephrine. A deficiency in the enzyme has not been associated with conduct disorder, but studies have shown an association between conduct disorder and certainly variations of the COMT gene.
- Lamotrigine, valproic acid, carbamazepine, and oxcarbazepine are cleared by?
the liver.
- At 12 weeks of SGA intake
the patient should have the following taken: weight with corresponding BMI, blood pressure, fasting plasma glucose, and fasting lipid profile. Annually, he or she should have a follow up of his or her personal/family history, waist circumference, blood pressure, and fasting plasma glucose.
- Guillain-Barre Syndrome (GBS) is a polyradiculoneuropathy which means it involves both the nerve roots of the spinal nerves and the peripheral nerves. As such
the presentation is consistent with lower motor neuron disease. GBS affects motor nerve function more prominently than sensory function, although sensory nerves are also affected. In GBS (as in most neuropathies other than diabetic neuropathy) bowel, bladder, and sexual function are preserved. GBS, however, does impair function of the phrenic and intercostal nerves and, thus, can lead to respiratory insufficiency or failure. It is treated with IV immunoglobins and plasma exchange.
What is power?
the probability of rejecting a false null hypothesis or the likelihood that the study will detect an effect of a treatment when the effect truly exists. Power of statistical significance is affected by the size of the effect and the sample size used.
- Cobalamin deficiency (vitamin B12) is characterized by the development of a chronic megaloblastic anemia (pernicious anemia). This results in neurological changes that include degenerative changes in the peripheral nerves
the spinal cord, and the brain. Neurological changes are seen in about 80% of all patients. Mental changes such as apathy, depression, irritability, and moodiness are common. In a few patients, encephalopathy will occur, as well as delirium, delusions, hallucinations, neurocognitive disorder, and sometimes paranoid features.
- Tasimelteon is the only drug approved for?
the treatment of Non-24 hour sleep-wake disorder by the US FDA
- According to the American Academy of Neurology (AAN) treatment guidelines The drug class of benzodiazepines can be used for?
the treatment of Tardive Dyskinesia
- According to guidelines how long should medication be continued after first MDE?
the treatment of the first episode of major depressive disorder should be continued for 9 months following a positive response.
- what is the risk for patients who have been hospitalized with major depressive disorder?
there is a 25% incidence of relapse within 6 months after discharge, a 30% to 50% incidence of recurrence in the first 2 years after discharge, and at least a 50% incidence of relapse after 5 years.
- In patients with anorexia nervosa
there is a decrease in the activity and turnover of norepinephrine. Therefore, there is a decrease in the levels of norepinephrine metabolite 3-methoxy-4-hydroxyphenylglycol (MHPG) in the urine and cerebrospinal fluid.
- Opioids cause suppression of norepinephrine release from LC neurons by inhibiting an enzyme that converts adenosine triphosphate (ATP) to cyclic adenosine monophosphate (cAMP). In patients experiencing symptoms of opioid withdrawal
there is an increase in the production of cAMP. Opioids bind to the mu receptors in locus ceruleus (LC) located at the base of the brain which results in suppression of norepinephrine release. Excessive amounts of norepinephrine is released in patients with opioid withdrawal as there are no opioids opposing the increased activity of LC neurons leading to production of symptoms. There is an increase in the activity of LC neurons due to absence of opioid inhibition. Opioids cause suppression of norepinephrine release from LC neurons by inhibiting an enzyme that converts adenosine triphosphate (ATP) to cyclic adenosine monophosphate (cAMP). In patients experiencing symptoms of opioid withdrawal, there is an increase in the production of cAMP. In patients experiencing symptoms of opioid withdrawal, there is a decrease in the levels of ATP as it is being converted into cAMP.
- In patients with schizophrenia experiencing cognitive impairment
there is decreased muscarinic and nicotinic receptors found in the caudate-putamen, hippocampus, and certain regions of prefrontal cortex.
- Serotonergic agents are used in the treatment of rejection sensitivity in patients with an avoidant personality disorder
they are less helpful autonomic symptoms than beta-blockers.
- Progressive multifocal leukoencephalopathy (PML) is caused by the opportunistic and ubiquitous JC virus (Polyomavirus JC) in immunodeficient individuals (e.g.
those with HIV/AIDs) and involves destruction of oligodendrocytes, which create myelin in the central nervous system.
- Phosphorylation is a posttranslational modification in which a phosphate group is added to serine
threonine or tyrosine on target proteins. This increases or decreases the functional activity of the protein.
- For PCP intoxication
treat agitation with lorazepam and psychosis with haloperidol.
- In the COMBINE (Combining Medications and Behavioral Interventions for Alcoholism) study
treatment with naltrexone was associated with an increase in percent days abstinent from drinking and a reduction in days of heavy drinking relative to placebo. Acamprosate was shown to be similar to placebo.
- N-acetylcysteine has demonstrated efficacy in treatment of what?
trichotillomania.
- Hyperalgesia refers to a heightened response to a painful or noxious stimulus and inflammatory hyperalgesia has been shown to have a protective biological role
unlike allodynia which is painful even to moderate stimuli like a bed sheet.
- The correct response is circadian rhythm sleep-wake disorder (CRSD) delayed sleep phase type. CRSDs occur when there is an inability to maintain alignment between the internal clock and the external environment. Delayed sleep-wake phase disorder (DSWPD) is characterized by an inability to fall asleep before early morning
usually after 2am, which can be more than 3 hours later than the conventional sleep time of the general population. This results in a delay in waking up the subsequent morning and excessive daytime sleepiness when they are unable to sleep in due to work, education or social requirements. However, if they are able sleep in, their sleep is of normal duration and quality. Chronotherapy is a behavioral technique where individuals delay their sleep times successively for 3 hours every day until they reach the sleep window they desire. In insomnia, patients experience poor sleep quality or quantity with difficulty falling asleep, staying asleep or waking up early in the morning. Chronotherapy does not affect the quality of sleep and is not used to treat insomnia.
- There is a reduction in the volume of amygdala in patients with schizophrenia on neuroimaging associated with cortical atrophy
ventricular enlargement, reduced volume of hippocampus, thalamus, parahippocampal gyrus and reduced cell size of the Purkinje cells in the cerebellum. There is also a decrease in cortical thickness.
- Among the most common neurological complications of AIDS are the AIDS dementia complex (sometimes also called HIV-associated dementia)
which causes symptoms such as encephalitis (inflammation of the brain), behavioral changes, and a gradual decline in cognitive function.
- Whipple's disease is caused by the bacterium Tropheryma whipplei
which creates a malabsorption syndrome, as well as skin rashes, diarrhea, arthritis, and cognitive decline. It can be successfully treated with a long course of antibiotics, such as doxycycline and hydroxychloroquine. Hypothyroidism is associated with fatigue, decreased libido, memory impairment, and irritability. Niacin deficiency or pellagra is associated with skin rash and diarrhea, but not malabsorption or arthritis.
- Research has demonstrated that both plasma and platelet levels of serotonin fluctuate during a migraine attack. This suggests that serotonin may be involved in the pathogenesis of migraine
which has led to the serotonergic abnormalities hypothesis. According to this hypothesis, when platelets are activated, they aggregate and release serotonin, thus increasing the plasma serotonin level. Accordingly, an increase in plasma serotonin level would be expected to cause vasoconstriction, whereas a decrease in serotonin would promote vasodilation. Platelet serotonin levels may drop precipitously during the headache phase of migraine while urine levels of serotonin and its metabolites rise during headaches. This suggests that there is a large release of serotonin during such attacks. In addition, drugs such as reserpine that cause the release and depletion of serotonin from tissue storage sites may precipitate migraine headaches, thus adding credibility to this hypothesis.
- Interpersonal or social rhythm therapy is a type of behavioral therapy
which helps in regulating circadian rhythm in patients with bipolar disorder by encouraging them to adapt a healthy routine by identifying their daily day to day moods, sleeping patterns, and eating habits etc.
- Patients with suicidal behavior have reduced serotonin activity in the central nervous system. 5-hydroxyindoleacetic acid (5-HIAA) is a metabolite of serotonin which is found in low levels in cerebrospinal fluid (CSF). Many studies have found low level of serotonin or 5-HIAA in the frontal cortex or brainstem in individuals who committed suicide. There are also significant changes in the serotonin receptors in these patients. There is an increase in the 5-HT1A and 5-HT2A serotonin receptors subtypes as well as increase in the protein expression of 5-HT2A receptor in the prefrontal cortex and hippocampus
which is a compensatory mechanism of the body to decreased serotonin activity of the neurons.
- Cyproheptadine blocks 5-HT1A and 5-HT2A receptors. It is used in the management of serotonin syndrome
which is characterized by hyperthermia, hyperreflexia, hypertension, diaphoresis, and altered mental status caused by excessive serotonergic activity.
- The HAM-D has been criticized for giving too much weight to physical or neurovegetative symptoms
which may not be indicative of major depressive disorder in elderly or medically ill patients.
- A significant minority of patients with seizure disorder develop psychotic symptoms
which may occur interictally, postictally, or (chronically) interictally. A family history of psychosis, younger age of onset of seizures, borderline intellectual function, and presence of complex partial seizures are correlated with the development of chronic interictal psychosis in patients with epilepsy.
- 30% of HIV patients suffer from axonal polyneuropathy
which typically presents with painful burning sensations and paresthesias.
- what is the timing for psychotic disorders to be included as "with peri-partum onset?
while the patient is pregnant or within four weeks of delivery.
- The predominant endocannabinoid receptors in the brain are CBR1 (cannabinoid receptor 1)
with the presence of CBR2 (cannabinoid receptor 2) in parts of cerebellum and brainstem, but their overall abundance is in the gastrointestinal tract and the immune system. Two primary endogenous receptor ligands that bind these two cannabinoid receptors have been identified. The first one is anandamide, also known as N-arachidonoylethanolamine (AEA) and the second is 2-arachidonoylglycerol (2-AG). Anandamide (AEA) acts as a partial agonist at CBR1 receptor with high-affinity, but because 2-AG acts as a full agonist at both CBR1 and CBR2 with mild-to-moderate affinity, it is considered to be more efficient in executing effects like the reward, addiction, anxiety, coordination, appetite, learning, sleep, and cognition. The CB1R is also involved in physiological and pathological conditions in the peripheral nervous system and peripheral tissues, including pain, energy metabolism, cardiovascular functions, inflammation, and liver functions. Cannabis (especially tetrahydrocannabinol [THC] and cannabidiol [CBD]) displaces these endogenous ligands from their receptors and brings about enhanced and prolonged effects that are apparent from acute and chronic cannabis use.
- The normal masculinization of the fetus by dihydrotestosterone (DHT) would not be achieved and may eventually lead to some developmental abnormalities including hypospadias. These side effects occur because dutasteride belongs to 5 alpha reductase inhibitor drugs which reduce conversion of progesterone to allopregnanolone and testosterone to dihydrotestosterone (DHT). Thus
women of childbearing age, such as in this case should be well aware of this side effect and hence avoid getting pregnant while on this drug or even 6 months after stopping its use.
- CPT involves three main phases including psychoeducation and identification of 'stuck points' or unhelpful beliefs regarding the trauma
written exposure component to challenge patient's beliefs, and further challenging 'stuck points' and reshaping them into positive thought patterns.
- In Wilson's disease (WD)
you will see decreased serum ceruloplasmin, decreased serum copper, increased urine copper- less than 20mg/dl while normal levels are 20-40mg/dl.
- Three types of classical opioid receptors exist:
µ (mu) or MOR, κ (kappa) or KOR, and δ (delta) or DOR. Another opioid-like receptor which has been identified is nociceptin or orphanin FQ peptide receptor or NOR, which activates the ORL-1 (opioid receptor like-1). The classical receptors are activated by endogenous opioids like encephalins, endorphins and dynorphins, whereas exogenous opioids, like morphine, act mostly on the mu receptors. This makes the mu receptors family most clinically important as the inhibition of nociceptive pathways exploited by the exogenous opioids are the prime responsibility of these receptors.